Exam 2

¡Supera tus tareas y exámenes ahora con Quizwiz!

When teaching a client recently diagnosed with type 1 diabetes mellitus, the client states, "I will never be able to stick myself with a needle." How would the nurse respond? a. "I can give your injections to you while you are here in the hospital." b. "Everyone gets used to giving themselves injections. It really does not hurt." c. "Your disease will not be managed properly if you refuse to administer the shots." d. "Tell me what it is about the injections that are concerning you."

"Tell me what it is about the injections that are concerning you."

A nurse reviews the laboratory test values for a client with a new diagnosis of diabetes mellitus type 2. Which A1C value would the nurse expect? a. 5.0% b. 5.7% c. 6.2% d. 7.4%

7.4%

A nurse assesses clients who are at risk for diabetes mellitus. Which client is at greatest risk? a. A 19-year-old Caucasian b. A 22-year-old African American c. A 44-year-old Asian American d. A 58-year-old American Indian

A 58-year-old American Indian

A nurse teaches a client who has COPD. Which statements related to nutrition would the nurse include in this client's teaching? (SATA) a. Avoid drinking fluids just before and during meals b. Rest before meals if you have dyspnea c. Have about six small meals a day d. Eat high-fiber foods to promote gastric emptying e. Use pursed-lip breathing during meals f. Choose soft, high calorie, high protein foods

A, B, C, E, F Clients with COPD often are malnourished for several reasons. The nurse would teach the client not to drink fluids before and with meals to avoid early satiety. The client needs rest before eating, and eat smaller frequent meals: 4 to 6 a day. Pursed-lip breathing will help control dyspnea. Food that is easy to eat will be less tiring and the client should choose high-calorie, high-protein foods.

15. A nurse cares for a client with infective endocarditis. Which infection control precautions should the nurse use? a. Standard Precautions b. Bleeding precautions c. Reverse isolation d. Contact isolation

ANS: A The client with infective endocarditis does not pose any specific threat of transmitting the causative organism. Standard Precautions should be used. Bleeding precautions or reverse or contact isolation is not necessary. DIF: Applying/Application REF: 697 KEY: Infection| Standard Precautions MSC: Integrated Process: Nursing Process: Implementation NOT: Client Needs Category: Safe and Effective Care Environment: Safety and Infection Control

11. A nurse cares for a client who is prescribed magnetic resonance imaging (MRI) of the heart. The client's health history includes a previous myocardial infarction and pacemaker implantation. What action would the nurse take? a. Schedule an electrocardiogram just before the MRI. b. Notify the primary health care provider before scheduling the MRI. c. Request lab for cardiac enzymes from the primary health care provider. d. Instruct the client to increase fluid intake the day before the MRI.

ANS: B The magnetic fields of the MRI can deactivate the pacemaker. The nurse would call the primary health care provider and report that the client has a pacemaker so that he or she can order other diagnostic tests. The client does not need an electrocardiogram, cardiac enzymes, or increased fluids. Some newer MRI scanners have eliminated the possibility of complications due to implants, but the nurse needs to notify the primary health care provider.

2. A nurse assesses a client in an outpatient clinic. Which statement alerts the nurse to the possibility of left-sided heart failure? a. "I have been drinking more water than usual." b. "I am awakened by the need to urinate at night." c. "I must stop halfway up the stairs to catch my breath." d. "I have experienced blurred vision on several occasions."

ANS: C Clients with left-sided heart failure report weakness or fatigue while performing normal activities of daily living, as well as difficulty breathing, or "catching their breath." This occurs as fluid moves into the alveoli. Nocturia is often seen with right-sided heart failure. Thirst and blurred vision are not related to heart failure. DIF: Understanding/Comprehension REF: 682 KEY: Heart failure| assessment/diagnostic examination MSC: IntegratedProcess:NursingProcess:Assessment NOT: Client Needs Category: Health Promotion and Maintenance

6. A student nurse is providing tracheostomy care. What action by the student requires intervention by the instructor? a. Holding the device securely when changing ties b. Suctioning the client first if secretions are present c. Tying a square knot at the back of the neck d. Using half-strength peroxide for cleansing

ANS: C To prevent pressure ulcers and for client safety, when ties are used that must be knotted, the knot should be placed at the side of the client's neck, not in back. The other actions are appropriate.

A nurse cares for a client experiencing diabetic ketoacidosis who presents with Kussmaul respirations. What action would the nurse take? a. Administration of oxygen via facemask b. Intravenous administration of 10% glucose c. Implementation of seizure precautions d. Administration of intravenous insulin

Administration of intravenous insulin

The nurse assesses a client with diabetic ketoacidosis. Which assessment finding would the nurse correlate with this condition? a. Increased rate and depth of respiration b. Extremity tremors followed by seizure activity c. Oral temperature of 102° F (38.9° C) d. Severe orthostatic hypotension

Increased rate and depth of respiration

A nurse assesses a client with diabetes mellitus who self-administers subcutaneous insulin. The nurse notes a spongy, swelling area at the site the client uses most frequently for insulin injection. What action would the nurse take? a. Apply ice to the site to reduce inflammation. b. Consult the provider for a new administration route. c. Assess the client for other signs of cellulitis. d. Instruct the client to rotate sites for insulin injection.

Instruct the client to rotate sites for insulin injection.

A nurse reviews the laboratory results of a client who is receiving intravenous insulin. Which would alert the nurse to intervene immediately? a. Serum chloride level of 98 mEq/L (98 mmol/L) b. Serum calcium level of 8.8 mg/dL (2.2 mmol/L) c. Serum sodium level of 132 mEq (132 mmol/L) d. Serum potassium level of 2.5 mEq/L (2.5 mmol/L)

Serum potassium level of 2.5 mEq/L (2.5 mmol/L)

A nurse cares for a patient who is prescribed pioglitazone. After 6 months of therapy, the client reports that he has a new onset of ankle edema. What assessment question would the nurse take? a. "Have you gained unexpected weight this week?" b. "Has your urinary output declined recently?" c. "Have you had fever and achiness this week?" d. "Have you had abdominal pain recently?"

"Have you gained unexpected weight this week?"

After teaching a patient with type 2 diabetes mellitus who is prescribed nateglinide, the nurse assesses the client's understanding. Which statement made by the patient indicates a correct understanding of the prescribed therapy? a. "I'll take this medicine during each of my meals." b. "I must take this medicine in the morning when I wake." c. "I will take this medicine before I go to bed." d. "I will take this medicine immediately before I eat."

"I will take this medicine immediately before I eat."

A nurse teaches a client with diabetes mellitus about foot care. Which statements would the nurse include in this client's teaching? (Select all that apply.) a. "Do not walk around barefoot." b. "Soak your feet in a tub each evening." c. "Trim toenails straight across with a nail clipper." d. "Treat any blisters or sores with Epsom salts." e. "Wash your feet every other day."

- "Do not walk around barefoot." - "Trim toenails straight across with a nail clipper."

A nurse collaborates with the interprofessional team to develop a plan of care for a client who is newly diagnosed with diabetes mellitus. Which team members would the nurse include in this interprofessional team meeting? (Select all that apply.) a. Registered dietitian nutritionist b. Clinical pharmacist c. Occupational therapist d. Primary health care provider e. Speech-language pathologist

- Registered dietitian nutritionist - Clinical pharmacist - Primary health care provider

A nurse provides diabetic education at a public health fair. Which disorders would the nurse include as complications of diabetes mellitus? (Select all that apply.) a. Stroke b. Kidney failure c. Blindness d. Respiratory failure e. Cirrhosis

- Stroke - Kidney failure - Blindness

A client in the emergency department reports difficulty breathing. The nurse assesses the clients appearance as depicted below: What action by the nurse is the priority? A. Assess blood pressure and pulse. B. Attach the client to a pulse oximeter. C. Have the client rate his or her pain. D. Start high-dose steroid therapy.

A. Assess blood pressure and pulse. Rationale: This client has superior vena cava syndrome, in which venous return from the head, neck, and trunk is blocked. Decreased cardiac output can occur. The nurse should assess indicators of cardiac output, including blood pressure and pulse, as the priority. The other actions are also appropriate but are not the priority.

5. An emergency department nurse obtains the health history of a client. Which statement by the client would alert the nurse to the occurrence of heart failure? a. "I get short of breath when I climb stairs." b. "I see halos floating around my head." c. "I have trouble remembering things." d. "I have lost weight over the past month."

ANS: A Dyspnea on exertion is an early manifestation of heart failure and is associated with an activity such as stair climbing. The other findings are not specific to early occurrence of heart failure.

19. The patient has right-sided heart failure. Which finding will the nurse expect when performing an assessment? a. Peripheral edema b. Basilar crackles c. Chest pain d. Cyanosis

ANS: A Right-sided heart failure results from inability of the right side of the heart to pump effectively, leading to a systemic backup. Peripheral edema, distended neck veins, and weight gain are signs of right-sided failure. Basilar crackles can indicate pulmonary congestion from left-sided heart failure. Cyanosis and chest pain result from inadequate tissue perfusion.

46. The nurse is caring for a patient who needs oxygen via a nasal cannula. Which task can the nurse delegate to the nursing assistive personnel? a. Applying the nasal cannula b. Adjusting the oxygen flow c. Assessing lung sounds d. Setting up the oxygen

ANS: A The skill of applying (not adjusting oxygen flow) a nasal cannula or oxygen mask can be delegated to nursing assistive personnel (NAP). The nurse is responsible for assessing the patient's respiratory system, response to oxygen therapy, and setup of oxygen therapy, including adjustment of oxygen flow rate.

5. A nurse is teaching a client about possible complications and hazards of home oxygen therapy. About which complications does the nurse plan to teach the client? (Select all that apply.) a. Absorptive atelectasis b. Combustion c. Dried mucous membranes d. Oxygen-induced hyperventilation e. Toxicity

ANS: A, B, C, E Complications of oxygen therapy include absorptive atelectasis, combustion, dried mucous membranes, and oxygen toxicity. Oxygen-induced hypoventilation is also a complication.

9. A nurse is caring for a client who was admitted with hypertrophic cardiomyopathy (HCM). What interprofessional care does the nurse anticipate providing? (Select all that apply.) a. Administering beta blockers b. Administering high-dose furosemide c. Preparing for a cardiac catheterization d. Loading the client on digitalis e. Instructing the client to avoid strenuous exercise f. Teaching the client how to use the CardioMEMS™

ANS: A, C, E

2. A hospital nurse is participating in a drill during which many "clients" with inhalation anthrax are being admitted. What drugs would the nurse anticipate administering? (Select all that apply.) a. Vancomycin b. Ciprofloxacin c. Doxycycline d. Ethambutol e. Sulfamethoxazole-trimethoprim (SMX-TMP)

ANS: A,B,C Vancomycin, ciprofloxacin, and doxycycline are all possible treatments for inhalation anthrax. Ethambutol is used for tuberculosis. SMX-TMP is commonly used for urinary tract infections and other common infections.

2. An emergency department nurse assesses a female client. Which assessment findings would alert the nurse to request a prescription for an electrocardiogram? (Select all that apply.) a. Hypertension b. Fatigue despite adequate rest c. Indigestion d. Abdominal pain e. Shortness of breath

ANS: B,C,E Women may not have chest pain with myocardial infarction, but may feel discomfort or indigestion. They often present with a triad of symptoms—indigestion or feeling of abdominal fullness, feeling of chronic fatigue despite adequate rest, and feeling unable to catch their breath. Frequently, women are not diagnosed and therefore are not treated adequately. Hypertension and abdominal pain are not associated with acute coronary syndrome.

13. A patient has heart failure and cardiac output is decreased. Which formula can the nurse use to calculate cardiac output? a. Myocardial contractility × Myocardial blood flow b. Ventricular filling time/Diastolic filling time c. Stroke volume × Heart rate d. Preload/Afterload

ANS: C Cardiac output can be calculated by multiplying the stroke volume and the heart rate. The other options are not measures of cardiac output.

A nurse cares for a client with diabetes mellitus who asks, "Why do I need to administer more than one injection of insulin each day?" How would the nurse respond? a. "You need to start with multiple injections until you become more proficient at self-injection." b. "A single dose of insulin each day would not match your blood insulin levels and your food intake patterns." c. "A regimen of a single dose of insulin injected each day would require that you eat fewer carbohydrates." d. "A single dose of insulin would be too large to be absorbed, predictably putting you at risk for insulin shock."

"A single dose of insulin each day would not match your blood insulin levels and your food intake patterns."

A nurse teaches a client with type 2 diabetes mellitus who is prescribed glipizide (Glucotrol). Which statement would the nurse include in this client's teaching? a. "Change positions slowly when you get out of bed." b. "Avoid taking nonsteroidal anti-inflammatory drugs (NSAIDs)." c. "If you miss a dose of this drug, you can double the next dose." d. "Discontinue the medication if you develop a urinary infection."

"Avoid taking nonsteroidal anti-inflammatory drugs (NSAIDs)."

The nurse is planning teaching for a client who is starting acarbose for diabetes mellitus type 2. Which statement will the nurse include in the teaching? a. "Be sure to take the drug with each meal." b. "Take the drug every evening before bedtime." c. "Take the drug on an empty stomach in the morning." d. "Decide on the best day of the week to take the drug."

"Be sure to take the drug with each meal."

After teaching a young adult client who is newly diagnosed with type 1 diabetes mellitus, the nurse assesses the client's understanding. Which statement made by the client indicates a correct understanding of the need for eye examinations? a. "At my age, I should continue seeing the ophthalmologist as I usually do." b. "I will see the eye doctor when I have a vision problem and yearly after age 40." c. "My vision will change quickly. I should see the ophthalmologist twice a year." d. "Diabetes can cause blindness, so I should see the ophthalmologist yearly."

"Diabetes can cause blindness, so I should see the ophthalmologist yearly."

A nurse teaches a patient about self-monitoring of blood glucose levels. Which statement would the nurse include in this client's teaching to prevent bloodborne infections? a. "Wash your hands after completing each test." b. "Do not share your monitoring equipment." c. "Blot excess blood from the strip with a cotton ball." d. "Use gloves when monitoring your blood glucose."

"Do not share your monitoring equipment."

The nurse is planning teaching for a client who is starting exenatide extended release (ER) for diabetes mellitus type 2. Which statement will the nurse include in the teaching? a. "Be sure to take the drug once a day before breakfast." b. "Take the drug every evening before bedtime." c. "Give your drug injection the same day every week." d. "Take the drug with dinner at the same time each day."

"Give your drug injection the same day every week."

A nurse develops a dietary plan for a client with diabetes mellitus and new-onset microalbuminuria. Which component of the client's diet would the nurse decrease? a. Carbohydrates b. Proteins c. Fats d. Total calories

Proteins

After teaching a client who is newly diagnosed with type 2 diabetes mellitus, the nurse assesses the client's understanding. Which statement made by the client indicates a need for additional teaching? a. "I should increase my intake of vegetables with higher amounts of dietary fiber." b. "My intake of saturated fats should be no more than 10% of my total calorie intake." c. "I should decrease my intake of protein and eliminate carbohydrates from my diet." d. "My intake of water is not restricted by my treatment plan or medication regimen."

"I should decrease my intake of protein and eliminate carbohydrates from my diet."

After teaching a client who has diabetes mellitus with retinopathy, nephropathy, and peripheral neuropathy, the nurse assesses the client's understanding. Which statement made by the client indicates a correct understanding of the teaching? a. "I have so many complications; exercising is not recommended." b. "I will exercise more frequently because I have so many complications." c. "I used to run for exercise; I will start training for a marathon." d. "I should look into swimming or water aerobics to get my exercise."

"I should look into swimming or water aerobics to get my exercise."

After teaching a client who is recovering from pancreas transplantation, the nurse assesses the client's understanding. Which statement made by the client indicates a need for further teaching? a. "If I develop an infection, I should stop taking my corticosteroid." b. "If I have pain over the transplant site, I will call the surgeon immediately." c. "I should avoid people who are ill or who have an infection." d. "I should take my cyclosporine exactly the way I was taught."

"If I develop an infection, I should stop taking my corticosteroid."

A nurse teaches a client who is diagnosed with diabetes mellitus. Which statement would the nurse include in this client's plan of care to delay the onset of microvascular and macrovascular complications? a. "Maintain tight glycemic control and prevent hyperglycemia." b. "Restrict your fluid intake to no more than 2 L a day." c. "Prevent hypoglycemia by eating a bedtime snack." d. "Limit your intake of protein to prevent ketoacidosis."

"Maintain tight glycemic control and prevent hyperglycemia."

A nurse teaches a client with diabetes mellitus about sick-day management. Which statement would the nurse include in this client's teaching? a. "When ill, avoid eating or drinking to reduce vomiting and diarrhea." b. "Monitor your blood glucose levels at least every 4 hours while sick." c. "If vomiting, do not use insulin or take your oral antidiabetic agent." d. "Try to continue your prescribed exercise regimen even if you are sick."

"Monitor your blood glucose levels at least every 4 hours while sick."

A nurse assesses several clients who have a history of respiratory disorders. Which client would the nurse assess FIRST? a. A 66 year old client with a barrel chest and clubbed fingernails b. A 48 year old client with an oxygen saturation level of 92% at rest c. A 35 year old client who reports orthopnea in bed d. A 27 year old client with a heart rate of 120 beats/min

D. A 27 year old client with a heartbeat of 120 beats/min Tachycardia can indicate hypoxemia as the body tries to circulate oxygen that is available. A barrel chest is not an emergency finding. Likewise, a pulse oximetry level of 92% is not considered an acute finding. Orthopnea at night in bed is breathlessness when lying down but is not an acute finding at this moment.

A nurse cares for a client who had a chest tube placed 6 hours ago and refuses to take deep breaths because of the pain. What action would the nurse take? a. Ambulate the client in the hallway to promote deep breathing b. Auscultate the client's anterior and posterior lung fields c. Encourage the client to take shallow breaths to help with the pain d. Administer pain medication and encourage the client to take deep breaths

D. Administer pain meds and encourage the client to take deep breaths A chest tube is paced in the pleural space and may be uncomfortable for the client. The nurse would provide pain medication to minimize discomfort and encourage the client to take deep breaths. Other responses do not address the client's discomfort and need to take deep breaths to prevent complications.

A nurse evaluates the following arterial blood gas and vital sign results for a client with COPD: ABG RESULTS: pH = 7.32; PaCO2 = 62 mmHg; PaO2 = 46 mmHg; HCO3 = 28 mEq/L VITAL SIGNS: HR = 110 beats/min; RR = 12 breaths/min; BP = 145/65; O2 saturation = 76% What action would the nurse take FIRST? a. Administer a short-acting beta2 agonist inhaler b. Document the findings as normal for a client with COPD c. Teach the client diaphragmatic breathing techniques d. Initiate oxygenation therapy to increase saturation to 88-92%

D. Initiate oxygenation therapy to increase saturation to 88-92% Oxygen would be administered to a client who is hypoxic even if the client has COPD and is a carbon dioxide retainer. The other interventions do not address the client's hypoxia, which is the major issue. There is no indication that the client needs an inhaler. Diaphragmatic breathing techniques would not be taught to a client in distress. These findings are not normal for all clients with COPD.

The nurse has taught a client with cancer ways to prevent infection. What statement by the client indicates that more teaching is needed? A. I should take my temperature daily and when I don't feel well. B. I will wash my toothbrush in the dishwasher once a week. C. I wont let anyone share any of my personal items or dishes. D. It's alright for me to keep my pets and change the litter box.

D. It's alright for me to keep my pets and change the litter box. Rationale: Clients should wash their hands after touching their pets and should not empty or scoop the cat litter box. The other statements are appropriate for self-management.

A nurse cares for a client who has a pleural chest tube. What action would the nurse take to ensure safe use of this equipment? a. Strip the tubing to minimize clot formation and ensure patency b. Secure tubing junctions with clamps to prevent accidental disconnections c. Connect the chest tube to wall suction as prescribed by the primary health care provider d. Keep padded clamps at the bedside for use if the drainage system is interrupted

D. Keep padded clamps at the bedside for use if the drainage system is interrupted Padded clamps would be kept at the bedside for use if the drainage system becomes dislodged or is interrupted. The nurse would never strip the tubing. Tubing junctions would be taped, not clamped. Wall suction would be set at a level indicated by the device's manufacturer, not the primary healthcare provider.

A new nurse has been assigned a client who is in the hospital to receive iodine-131 treatment. Which action by the nurse is best? A. Ensure the client is placed in protective isolation. B. Hand off a pregnant client to another nurse. C. No special action is necessary to care for this client. D. Read the policy on handling radioactive excreta.

D. Read the policy on handling radioactive excreta. Rationale: This type of radioisotope is excreted in body fluids and excreta (urine and feces) and should not be handled directly. The nurse should read the facilitys policy for handling and disposing of this type of waste. The other actions are not warranted.

A nurse is assessing a female client who is taking progestins. What assessment finding requires the nurse to notify the provider immediately? A. Irregular menses B. Edema in the lower extremities C. Ongoing breast tenderness D. Red, warm, swollen calf

D. Red, warm, swollen calf Rationale: All clients receiving progestin therapy are at risk for thromboembolism. A red, warm, swollen calf is a manifestation of deep vein thrombosis and should be reported to the provider. Irregular menses, edema in the lower extremities, and breast tenderness are common side effects of the therapy.

A nurse works with clients who have alopecia from chemotherapy. What action by the nurse takes priority? A. Helping clients adjust to their appearance B. Reassuring clients that this change is temporary C. Referring clients to a reputable wig shop D. Teaching measures to prevent scalp injury

D. Teaching measures to prevent scalp injury Rationale: All of the actions are appropriate for clients with alopecia. However, the priority is client safety, so the nurse should first teach ways to prevent scalp injury.

A client with cancer is admitted to a short-term rehabilitation facility. The nurse prepares to administer the clients oral chemotherapy medications. What action by the nurse is most appropriate? A. Crush the medications if the client cannot swallow them. B. Give one medication at a time with a full glass of water. C. No special precautions are needed for these medications. D. Wear personal protective equipment when handling the medications.

D. Wear personal protective equipment when handling the medications. Rationale: During the administration of oral chemotherapy agents, nurses must take the same precautions that are used when administering IV chemotherapy. This includes using personal protective equipment. These medications cannot be crushed, split, or chewed. Giving one at a time is not needed.

The nurse is caring for a newly admitted client who is diagnosed with hyperglycemic-hyperosmolar state (HHS). What is the nurse's priority action at this time? a. Assess the client's blood glucose level. b. Monitor the client's urinary output every hour. c. Establish intravenous access to provide fluids. d. Give regular insulin per agency policy

Establish intravenous access to provide fluids.

A nurse assesses a client who has a 15-year history of diabetes and notes decreased tactile sensation in both feet. What action would the nurse take first? a. Document the finding in the client's chart. b. Assess tactile sensation in the client's hands. c. Examine the client's feet for signs of injury. d. Notify the primary health care provider.

Examine the client's feet for signs of injury.

A nurse reviews laboratory results for a client with diabetes mellitus who is prescribed an intensified insulin regimen: • Fasting blood glucose: 75 mg/dL (4.2 mmol/L) • Postprandial blood glucose: 200 mg/dL (11.1 mmol/L) • Hemoglobin A1C level: 5.5% How would the nurse interpret these laboratory findings? a. Increased risk for developing ketoacidosis b. Good control of blood glucose c. Increased risk for developing hyperglycemia d. Signs of insulin resistance

Good control of blood glucose

The nurse is caring for a newly admitted older adult who has a blood glucose of 300 mg/dL (16.7 mmol/L), a urine output of 185 mL in the past 8 hours, and a blood urea nitrogen (BUN) of 44 mg/dL (15.7 mmol/L). What diabetic complication does the nurse suspect? a. Diabetic ketoacidosis (DKA) b. Severe hypoglycemia c. Chronic kidney disease (CKD) d. Hyperglycemic-hyperosmolar state (HHS)

Hyperglycemic-hyperosmolar state (HHS)

A nurse assesses a client with diabetes mellitus. Which assessment finding would alert the nurse to decreased kidney function in this client? a. Urine specific gravity of 1.033 b. Presence of protein in the urine c. Elevated capillary blood glucose level d. Presence of ketone bodies in the urine

Presence of protein in the urine

A nurse cares for a client with arthritis who reports frequent asthma attacks. What action would the nurse take FIRST? a. Review the client's pulmonary function test results b. Ask about medications the client is currently taking c. Assess how frequently the client uses a bronchodilator c. Consult the primary healthcare provider and request arterial blood gases

b. Ask about medications the client is currently taking Aspirin and other NSAIDs can trigger asthma in some people. This results from increased production of leukotriene when aspirin or NSAIDs suppress inflammatory pathways and is a likely culprit given the client's history. Reviewing pulmonary function test results will not address the immediate problem of frequent asthma attacks. This is a good time to review response to bronchodilators, but assessing triggers is more important. Questioning the client about the use of bronchodilators will address interventions for the attacks but not their cause. Reviewing arterial blood gas results would not be of use in a client between attacks because many clients are asymptomatic when not having attacks.

A nurse assesses a client who has diabetes mellitus. Which arterial blood gas values would the nurse identify as potential ketoacidosis in this client? a. pH 7.38, HCO3 22 mEq/L (22 mmol/L), PCO2 38 mm Hg, PO2 98 mm Hg b. pH 7.28, HCO3 18 mEq/L (18 mmol/L), PCO2 28 mm Hg, PO2 98 mm Hg c. pH 7.48, HCO3 28 mEq/L (28 mmol/L), PCO2 38 mm Hg, PO2 98 mm Hg d. pH 7.32, HCO3 22 mEq/L (22 mmol/L), PCO2 58 mm Hg, PO2 88 mm Hg

pH 7.28, HCO3− 18 mEq/L (18 mmol/L), PCO2 28 mm Hg, PO2 98 mm Hg

After teaching a client with diabetes mellitus to inject insulin, the nurse assesses the client's understanding. Which statement made by the client indicates a need for further teaching? a. "The lower abdomen is the best location because it is closest to the pancreas." b. "I can reach my thigh the best, so I will use the different areas of my thighs." c. "By rotating the sites in one area, my chance of having a reaction is decreased." d. "Changing injection sites from the thigh to the arm will change absorption rates."

"The lower abdomen is the best location because it is closest to the pancreas."

A nurse teaches a client with diabetes mellitus who is experiencing numbness and reduced sensation. Which statement would the nurse include in this client's teaching to prevent injury? a. "Examine your feet using a mirror every day." b. "Rotate your insulin injection sites every week." c. "Check your blood glucose level before each meal." d. "Use a bath thermometer to test the water temperature."

"Use a bath thermometer to test the water temperature."

A nurse teaches a client with type 1 diabetes mellitus. Which statement would the nurse include in this client's teaching to decrease the client's insulin needs? a. "Limit your fluid intake to 2 L a day." b. "Animal organ meat is high in insulin." c. "Limit your carbohydrate intake to 80 g a day." d. "Walk at a moderate pace for 1 mile daily."

"Walk at a moderate pace for 1 mile daily."

A nurse is teaching a client with diabetes mellitus who asks, "Why is it necessary to maintain my blood glucose levels no lower than about 60 mg/dL (3.3 mmol/L)?" How would the nurse respond? a. "Glucose is the only fuel used by the body to produce the energy that it needs." b. "Your brain needs a constant supply of glucose because it cannot store it." c. "Without a minimum level of glucose, your body does not make red blood cells." d. "Glucose in the blood prevents the formation of lactic acid and prevents acidosis."

"Your brain needs a constant supply of glucose because it cannot store it."

A nurse cares for a client who has a family history of diabetes mellitus. The client states, "My father has type 1 diabetes mellitus. Will I develop this disease as well?" How would the nurse respond? a. "Your risk of diabetes is higher than the general population, but it may not occur." b. "No genetic risk is associated with the development of type 1 diabetes mellitus." c. "The risk for becoming a diabetic is 50% because of how it is inherited." d. "Female children do not inherit diabetes mellitus, but male children will."

"Your risk of diabetes is higher than the general population, but it may not occur."

A nurse assesses adults at a health fair. Which adults would the nurse counsel to be tested for diabetes? (Select all that apply.) a. A 56-year-old African-American male b. A 22-year-old female with a 30-lb (13.6 kg) weight gain during pregnancy c. A 60-year-old male with a history of liver trauma d. A 48-year-old female with a sedentary lifestyle e. A 50-year-old male with a body mass index greater than 25 kg/m2 f. A 28-year-old female who gave birth to a baby weighing 9.2 lb (4.2 kg)

- A 56-year-old African-American male - A 48-year-old female with a sedentary lifestyle - A 50-year-old male with a body mass index greater than 25 kg/m2 - A 28-year-old female who gave birth to a baby weighing 9.2 lb (4.2 kg)

The nurse is caring for a client who has severe hypoglycemia and is experiencing a seizure. What actions will the nurse take at this time? (Select all that apply.) a. Administer glucagon 1 mg subcutaneously. b. Be sure the bed side rails are in the up position. c. Notify the primary health care provider immediately. d. Monitor the client's blood glucose level. e. Increase the intravenous infusion rate immediately.

- Administer glucagon 1 mg subcutaneously. - Be sure the bed side rails are in the up position. - Notify the primary health care provider immediately. - Monitor the client's blood glucose level.

A nurse assesses a patient who is experiencing diabetic ketoacidosis (DKA). For which assessment findings would the nurse monitor the client? (Select all that apply.) a. Deep and fast respirations b. Decreased urine output c. Tachycardia d. Dependent pulmonary crackles e. Orthostatic hypotension

- Deep and fast respirations - Tachycardia - Orthostatic hypotension

The nurse is caring for a client who has diabetes mellitus. The nurse administers 6 units of regular insulin and 10 units of NPH insulin at 7:00 a.m. (0700). At which time would the nurse assess the client for potential hypoglycemia related to the NPH insulin? a. 8:00 a.m. (0800) b. 4:00 p.m. (1600) c. 8:00 p.m. (2000) d. 11:00 p.m. (2300)

4:00 p.m. (1600)

A client, who has become increasingly dyspneic over a year, has been diagnosed with pulmonary fibrosis. What information would the nurse plan to include in teaching this client? (SATA) a. The need to avoid large crowds and people who are ill b. Safety measures to take if home oxygen is needed c. Information about appropriate use of the drug nintedanib d. Genetic therapy to stop progression of the disease e. Measures to avoid fatigue during the day f. The possibility of receiving a lung transplant if infection-free for a year

A, B, C, E Pulmonary fibrosis is a progressive disorder with no cure. Therapy focuses on slowing progression and managing dyspnea. Clients need to avoid contracting infections so should be taught to stay away from large crowds and sick people. Home O2 is needed and the nurse would teach safety measures related to O2. The drug nintedanib has shown to improve cellular regulation and slow progression of the disease. Gene therapy is not available. Energy conservation measures are also an important topic. Lung transplantation is an unlikely option due to selection criteria.

A nurse plans care for a client who has COPD and thick, tenacious secretions. Which interventions would the nurse include in this client's plan of care? (SATA) a. Ask the client to drink 2 L of fluids daily b. Add humidity to the prescribed oxygen c. Suction the client every 2 to 3 hours d. Use a vibrating chest physiotherapy device e. Encourage diaphragmatic breathing f. Administer the ordered mucolytic agent

A, B, D, F Interventions to decrease thick, tenacious secretions include maintaining adequate hydration and providing humidified oxygen. These actions will help to thin secretions, making them easier to remove by coughing. The use of a vibrating chest physiotherapy device can also help clients remove thick secretions but is usually used in clients with cystic fibrosis. Mucolytic agents help thin secretions, making them easier to bring up. Although suctioning may assist with the removal of secretions, frequent suctioning can cause airway trauma and does not support the client's ability to successfully remove secretions through normal coughing. Diaphragmatic breathing is not used to improve the removal of thick secretions.

A nurse is assessing a client with lung cancer. What nonpulmonary signs and symptoms should the nurse be aware of? (SATA) a. Gynecomastia in male patients b. Frequent shaking and sweating relieved by eating c. Positive Chvostek and trousseau signs d. "Moon" face and "buffalo" hump e. Expectorating purulent sputum f. General edema

A, B, D, F Lung cancer is often associated with paraneoplastic syndrome. Symptoms of these include gynecomastia from ectopic follicle-stimulating hormone release, hypoglycemia from ectopic insulin production (shaking and sweating relieved by eating), and Cushing syndrome (moon face and buffalo hum) from ectopic adrenocorticotropic hormone. General edema can be caused by antidiuretic hormone.

A home health nurse evaluates a client who has COPD. Which assessments would the nurse include in this client's evaluation? (SATA) a. Examination of mucous membranes and nailbeds b. Measurement of rate, depth, and rhythm of respirations c. Auscultation of bowel sounds for abnormal sounds d. Check peripheral veins for distension while at rest e. Determine the client's need and use of oxygen f. Ability to perform activities of daily living

A, B, E, F A home health nurse would: assess the client's respiratory status and adequacy of ventilation, including an examination of mucous membranes and nail beds for evidence of hypoxia; measure the rate, depth, and rhythm of respirations; auscultate lung fields for abnormal breath sounds; check neck veins for distension when the client is in a sitting position; and determine the client's needs and use of supplemental oxygen. The home health nurse would also determine the client's ability to perform his or her own ADLs. Auscultation of bowel sounds and assessment of peripheral veins are not part of a focused assessment for a client with COPD.

A nurse cares for a client who is prescribed an intravenous prostacyclin agent for pulmonary artery hypertension. What actions would the nurse take to ensure the client's safety while on this medication? (SATA) a. Keep an intravenous line dedicated strictly to the infusion b. Teach the client that this medication increases pulmonary pressures c. Ensure that there is always a backup drug cassette available d. Start a large-bore peripheral intravenous line e. Use strict aseptic technique when using the drug delivery system

A, C, E IV prostacyclin agents would be administered to a client with pulmonary artery hypertension through a central venous catheter with a dedicated IV line for medication. Death has been reported when the drug delivery system is interrupted even briefly; therefore, a backup drug cassette would also be available. The nurse would use strict aseptic technique when using the drug delivery system. The nurse would teach the client this medication decreases pulmonary pressures and increases lung blood flow.

A nurse works on an oncology unit and delegates personal hygiene to an unlicensed assistive personnel (UAP). What action by the UAP requires intervention from the nurse? A. Allowing a very tired client to skip oral hygiene and sleep B. Assisting clients with washing the perianal area every 12 hours C. Helping the client use a soft-bristled toothbrush for oral care D. Reminding the client to rinse the mouth with water or saline

A. Allowing a very tired client to skip oral hygiene and sleep Rationale: Even though clients may be tired, they still need to participate in hygiene to help prevent infection. The other options are all appropriate.

A client on interferon therapy is reporting severe skin itching and irritation. What actions does the nurse delegate to the unlicensed assistive personnel (UAP)? (Select all that apply.) A. Apply moisturizers to dry skin. B. Apply steroid creams to the skin. C. Bathe the client using mild soap. D. Help the client with a hot water bath. E. Teach the client to avoid sunlight.

A. Apply moisturizers to dry skin. C. Bathe the client using mild soap. Rationale: The nurse can delegate applying unscented moisturizer and using mild soap for bathing. Steroid creams are not used for this condition. Hot water will worsen the irritation. Client teaching is a nursing function.

A client has thrombocytopenia. What actions does the nurse delegate to the unlicensed assistive personnel (UAP)? (Select all that apply.) A. Apply the clients shoes before getting the client out of bed. B. Assist the client with ambulation. C. Shave the client with a safety razor only. D. Use a lift sheet to move the client up in bed. E. Use the Waterpik on a low setting for oral care.

A. Apply the clients shoes before getting the client out of bed. B. Assist the client with ambulation. D. Use a lift sheet to move the client up in bed. Rationale: Clients with thrombocytopenia are at risk of significant bleeding even with minor injuries. The nurse instructs the UAP to put the clients shoes on before getting the client out of bed, assist with ambulation, shave the client with an electric razor, use a lift sheet when needed to reposition the client, and use a soft-bristled toothbrush for oral care.

A clients family members are concerned that telling the client about a new finding of cancer will cause extreme emotional distress. They approach the nurse and ask if this can be kept from the client. What actions by the nurse are most appropriate? (Select all that apply.) A. Ask the family to describe their concerns more fully. B. Consult with a social worker, chaplain, or ethics committee. C. Explain the clients right to know and ask for their assistance. D. Have the unit manager take over the care of this client and family. E. Tell the family that this secret will not be kept from the client.

A. Ask the family to describe their concerns more fully. B. Consult with a social worker, chaplain, or ethics committee. C. Explain the clients right to know and ask for their assistance. Rationale: The clients right of autonomy means that the client must be fully informed as to his or her diagnosis and treatment options. The nurse cannot ethically keep this information from the client. The nurse can ask the family to explain their concerns more fully so everyone understands the concerns. A social worker, chaplain, or ethics committee can become involved to assist the nurse, client, and family. The nurse should explain the clients right to know and ask the family how best to proceed. The nurse should not abdicate responsibility for this difficult situation by transferring care to another nurse. Simply telling the family that he or she will not keep this secret sets up an adversarial relationship. Explaining this fact along with the concept of autonomy would be acceptable, but this by itself is not.

A client receiving chemotherapy has a white blood cell count of 1000/mm3. What actions by the nurse are most appropriate? (Select all that apply.) A. Assess all mucous membranes every 4 to 8 hours. B. Do not allow the client to eat meat or poultry. C. Listen to lung sounds and monitor for cough. D. Monitor the venous access device appearance with vital signs. E. Take and record vital signs every 4 to 8 hours.

A. Assess all mucous membranes every 4 to 8 hours. C. Listen to lung sounds and monitor for cough. D. Monitor the venous access device appearance with vital signs. E. Take and record vital signs every 4 to 8 hours. Rationale: Depending on facility protocol, the nurse should assess this client for infection every 4 to 8 hours by assessing all mucous membranes, listening to lung sounds, monitoring for cough, monitoring the appearance of the venous access device, and recording vital signs. Eating meat and poultry is allowed.

A client with a history of prostate cancer is in the clinic and reports new onset of severe low back pain. What action by the nurse is most important? A. Assess the clients gait and balance. B. Ask the client about the ease of urine flow. C. Document the report completely. D. Inquire about the clients job risks.

A. Assess the clients gait and balance. Rationale: This client has manifestations of spinal cord compression, which can be seen with prostate cancer. This may affect both gait and balance and urinary function. For client safety, assessing gait and balance is the priority. Documentation should be complete. The client may or may not have occupational risks for low back pain, but with his history of prostate cancer, this should not be where the nurse starts investigating.

A client is receiving chemotherapy through a peripheral IV line. What action by the nurse is most important? A. Assessing the IV site every hour B. Educating the client on side effects C. Monitoring the client for nausea D. Providing warm packs for comfort

A. Assessing the IV site every hour Rationale: Intravenous chemotherapy can cause local tissue destruction if it extravasates into the surrounding tissues. Peripheral IV lines are more prone to this than centrally placed lines. The most important intervention is prevention, so the nurse should check hourly to ensure the IV site is patent, or frequently depending on facility policy. Education and monitoring for side effects such as nausea are important for all clients receiving chemotherapy. Warm packs may be helpful for comfort, but if the client reports that an IV site is painful, the nurse needs to assess further.

A client has mucositis. What actions by the nurse will improve the clients nutrition? (Select all that apply.) A. Assist with rinsing the mouth with saline frequently. B. Encourage the client to eat room-temperature foods. C. Give the client hot liquids to hold in the mouth. D. Provide local anesthetic medications to swish and spit. E. Remind the client to brush teeth gently after each meal.

A. Assist with rinsing the mouth with saline frequently. B. Encourage the client to eat room-temperature foods. D. Provide local anesthetic medications to swish and spit. E. Remind the client to brush teeth gently after each meal. Rationale: Mucositis can interfere with nutrition. The nurse can help with rinsing the mouth frequently with water or saline; encouraging the client to eat cool, slightly warm, or room-temperature foods; providing swish-and-spit anesthetics; and reminding the client to keep the mouth clean by brushing gently after each meal. Hot liquids would be painful for the client.

A nurse working with clients who experience alopecia knows that which is the best method of helping clients manage the psychosocial impact of this problem? A. Assisting the client to pre-plan for this event B. Reassuring the client that alopecia is temporary C. Teaching the client ways to protect the scalp D. Telling the client that there are worse side effects

A. Assisting the client to pre-plan for this event Rationale: Alopecia does not occur for all clients who have cancer, but when it does, it can be devastating. The best action by the nurse is to teach the client about the possibility and to give the client multiple choices for preparing for this event. Not all clients will have the same reaction, but some possible actions the client can take are buying a wig ahead of time, buying attractive hats and scarves, and having a hairdresser modify a wig to look like the clients own hair. Teaching about scalp protection is important but does not address the psychosocial impact. Reassuring the client that hair loss is temporary and telling him or her that there are worse side effects are both patronizing and do not give the client tools to manage this condition.

A client is receiving interleukins along with chemotherapy. What assessment by the nurse takes priority? A. Blood pressure B. Lung assessment C. Oral mucous membranes D. Skin integrity

A. Blood pressure Rationale: Interleukins can cause capillary leak syndrome and fluid shifting, leading to intravascular volume depletion. Although all assessments are important in caring for clients with cancer, blood pressure and other assessments of fluid status take priority.

A nurse in the oncology clinic is providing preoperative education to a client just diagnosed with cancer. The client has been scheduled for surgery in 3 days. What action by the nurse is best? A. Call the client at home the next day to review teaching. B. Give the client information about a cancer support group. C. Provide all the preoperative instructions in writing. D. Reassure the client that surgery will be over soon.

A. Call the client at home the next day to review teaching. Rationale: Clients are often overwhelmed at a sudden diagnosis of cancer and may be more overwhelmed at the idea of a major operation so soon. This stress significantly impacts the clients ability to understand, retain, and recall information. The nurse should call the client at home the next day to review the teaching and to answer questions. The client may or may not be ready to investigate a support group, but this does not help with teaching. Giving information in writing is important (if the client can read it), but in itself will not be enough. Telling the client that surgery will be over soon is giving false reassurance and does nothing for teaching.

A nurse is preparing to administer IV chemotherapy. What supplies does this nurse need? (Select all that apply.) A. Chemo gloves B. Facemask C. Isolation gown D. N95 respirator E. Shoe covers

A. Chemo gloves B. Facemask C. Isolation gown Rationale: The Occupational Safety and Health Administration (OSHA) and the Oncology Nurses Society have developed safety guidelines for those preparing or administering IV chemotherapy. These include double gloves (or chemo gloves), a facemask, and a gown. An N95 respirator and shoe covers are not required.

A nurse reads on a hospitalized clients chart that the client is receiving teletherapy. What action by the nurse is best? A. Coordinate continuation of the therapy. B. Place the client on radiation precautions. C. No action by the nurse is needed at this time. D. Restrict visitors to only adults over age 18.

A. Coordinate continuation of the therapy. Rationale: The client needs to continue with radiation therapy, and the nurse can coordinate this with the appropriate department. The client is not radioactive, so radiation precautions and limiting visitors are not necessary.

The nurse working with oncology clients understands that which age-related change increases the older clients susceptibility to infection during chemotherapy? A. Decreased immune function B. Diminished nutritional stores C. Existing cognitive deficits D. Poor physical reserves

A. Decreased immune function Rationale: As people age, there is an age-related decrease in immune function, causing the older adult to be more susceptible to infection than other clients. Not all older adults have diminished nutritional stores, cognitive dysfunction, or poor physical reserves.

A nurse assesses a client who is prescribed fluticasone and notes oral lesions. What action would the nurse take? a. Encourage oral rinsing after fluticasone administration b. Obtain an oral specimen for culture and sensitivity c. Start the client on a broad-spectrum antibiotic d. Document the finding as a known side effect

A. Encourage oral rinsing after fluticasone administration The drug reduces local immunity and increases the risk for local infection, especially Candida albicans. Rinsing the mouth after using the inhaler will decrease the risk for developing this infection. Use of mouthwash and broad-spectrum antibiotics is not warranted in this situation. The nurse would document the finding, but the best action to take is to have the client start rinsing his or her mouth after using fluticasone. An oral specimen for culture and sensitivity is not necessary to care for this client.

A client hospitalized for chemotherapy has a hemoglobin of 6.1 mg/dL. What medication should the nurse prepare to administer? A. Epoetin alfa (Epogen) B. Filgrastim (Neupogen) C. Mesna (Mesnex) D. Oprelvekin (Neumega)

A. Epoetin alfa (Epogen) Rationale: The clients hemoglobin is low, so the nurse should prepare to administer epoetin alfa, a colony-stimulating factor that increases production of red blood cells. Filgrastim is for neutropenia. Mesna is used to decrease bladder toxicity from some chemotherapeutic agents. Oprelvekin is used to increase platelet count.

A nurse cares for a client with a 40 year smoking history who is experiencing distended neck veins and dependent edema. Which physiologic process would the nurse correlate with this client's history and clinical signs and symptoms? a. Increased pulmonary pressure creating higher workload on the right side of the heart b. Exposure to irritants resulting in increased inflammation fo the bronchi and bronchioles c. Increased number and size of mucous glands producing large amounts of thick mucus d. Left ventricular hypertrophy creating a decrease in cardiac output

A. Increased pulmonary pressure creating higher workload on the right side of the heart Smoking increases pulmonary hypertension, resulting in cor pulmonale, or right-sided heart failure. Increased pressures in the lungs make it more difficult for blood to flow through the lungs. Blood backs up into the right side of the heart and then into the peripheral venous system, creating distended neck veins and dependent edema. Inflammation in bronchi and bronchioles creates an airway obstruction which manifests as wheezes. Thick mucus in the lungs has no impact on distended neck veins and edema. Left ventricular hypertrophy is associated with left-sided heart failure and is not directly caused by a 40 year smoking history.

The nurse caring for oncology clients knows that which form of metastasis is the most common? a. Bloodborne b. Direct invasion c. Lymphatic spread d. Via bone marrow

ANS: A Bloodborne metastasis is the most common way for cancer to metastasize. Direct invasion and lymphatic spread are other methods. Bone marrow is not a medium in which cancer spreads, although cancer can occur in the bone marrow.

A client asks the nurse if eating only preservative- and dye-free foods will decrease cancer risk. What response by the nurse is best? a. "Maybe; preservatives, dyes, and preparation methods may be risk factors." b. "No; research studies have never shown those things to cause cancer." c. "There are other things you can do that will more effectively lower your risk." d. "Yes; preservatives and dyes are well known to be carcinogens."

ANS: A Dietary factors related to cancer development are poorly understood, although dietary practices are suspected to alter cancer risk. Suspected dietary risk factors include low-fiber intake and a high intake of red meat or animal fat. Preservatives, preparation methods, and additives (dyes, flavorings, sweeteners) may have cancer-promoting effects. It is correct to say that other things can lower risk more effectively, but this does not give the client concrete information about how to do so, and also does not answer the client's question.

27. A nurse is caring for a patient who has poor tissue perfusion as the result of hypertension. When the patient asks what to eat for breakfast, which meal should the nurse suggest? a. A cup of nonfat yogurt with granola and a handful of dried apricots b. Whole wheat toast with butter and a side of bacon c. A bowl of cereal with whole milk and a banana d. Omelet with sausage, cheese, and onions

ANS: A A 2000-calorie diet of fruits, vegetables, and low-fat dairy foods that are high in fiber, potassium, calcium, and magnesium and low in saturated and total fat helps prevent and reduce the effects of hypertension. Nonfat yogurt with granola is a good source of calcium, fiber, and potassium; dried apricots add a second source of potassium. Although cereal and a banana provide fiber and potassium, skim milk should be substituted for whole milk to decrease fat. An omelet with sausage and cheese is high in fat. Butter and bacon are high in fat.

25. A patient has carbon dioxide retention from lung problems. Which type of diet will the nurse most likely suggest for this patient? a. Low-carbohydrate b. Low-caffeine c. High-caffeine d. High-carbohydrate

ANS: A A low-carbohydrate diet is best. Diets high in carbohydrates play a role in increasing the carbon dioxide load for patients with carbon dioxide retention. As carbohydrates are metabolized, an increased load of carbon dioxide is created and excreted via the lungs. A low- or high-caffeine diet is not as important as the carbohydrate load.

18. A nurse admits a client from the emergency department. Client data are listed below: History Physical Assessment Laboratory Values What action by the nurse is the priority? ° 70 years of age ° History of diabetes ° On insulin twice a day ° Reports new onset dyspnea and productive cough ° Crackles and rhonchi heard throughout the lungs ° Dullness to percussion LLL ° Afebrile ° Oriented to person only ° WBC 5,200/mm3 (5.2 109/L) ° PaO2 on room air 85 mm Hg a. Administer oxygen at 4 L per nasal cannula. b. Begin broad-spectrum antibiotics. c. Collect a sputum sample for culture. d. Start an IV of normal saline at 50 mL/hr.

ANS: A All actions are appropriate for this client who has signs and symptoms of pneumonia. However, airway and breathing come first, so begin oxygen administration and titrate it to maintain saturations greater than 95%. Start the IV and collect a sputum culture, and then begin antibiotics.

1. A nurse assesses clients on a cardiac unit. Which client should the nurse identify as being at greatest risk for the development of left-sided heart failure? a. A 36-year-old woman with aortic stenosis b. A 42-year-old man with pulmonary hypertension c. A 59-year-old woman who smokes cigarettes daily d. A 70-year-old man who had a cerebral vascular accident

ANS: A Although most people with heart failure will have failure that progresses from left to right, it is possible to have left-sided failure alone for a short period. It is also possible to have heart failure that progresses from right to left. Causes of left ventricular failure include mitral or aortic valve disease, coronary artery disease, and hypertension. Pulmonary hypertension and chronic cigarette smoking are risk factors for right ventricular failure. A cerebral vascular accident does not increase the risk of heart failure. DIF: Applying/Application REF: 679 KEY: Heart failure| health screening MSC: IntegratedProcess:NursingProcess:Assessment NOT: Client Needs Category: Safe and Effective Care Environment: Management of Care

34. The patient is experiencing angina pectoris. Which assessment finding does the nurse expect when conducting a history and physical examination? a. Experiences chest pain after eating a heavy meal b. Experiences adequate oxygen saturation during exercise c. Experiences crushing chest pain for more than 20 minutes d. Experiences tingling in the left arm that lasts throughout the morning

ANS: A Angina pectoris is chest pain that results from limited oxygen supply. Often pain is precipitated by activities such as exercise, stress, and eating a heavy meal and lasts 3 to 5 minutes. Symptoms of angina pectoris are relieved by rest and/or nitroglycerin. Adequate oxygen saturation occurs with rest; inadequate oxygen saturation occurs during exercise. Pain lasting longer than 20 minutes or arm tingling that persists could be a sign of myocardial infarction.

6. A nurse is teaching a client with heart failure who has been prescribed enalapril (Vasotec). Which statement should the nurse include in this client's teaching? a. "Avoid using salt substitutes." b. "Take your medication with food." c. "Avoid using aspirin-containing products." d. "Check your pulse daily."

ANS: A Angiotensin-converting enzyme (ACE) inhibitors such as enalapril inhibit the excretion of potassium. Hyperkalemia can be a life-threatening side effect, and clients should be taught to limit potassium intake. Salt substitutes are composed of potassium chloride. ACE inhibitors do not need to be taken with food and have no impact on the client's pulse rate. Aspirin is often prescribed in conjunction with ACE inhibitors and is not contraindicated. DIF: Applying/Application REF: 685 KEY: Heart failure| angiotensin-converting enzyme (ACE) inhibitor| medication| patient education MSC: IntegratedProcess:Teaching/Learning NOT: Client Needs Category: Physiological Integrity: Pharmacological and Parenteral Therapies

11. A nurse admits a client who is experiencing an exacerbation of heart failure. Which action should the nurse take first? a. Assess the client's respiratory status. b. Draw blood to assess the client's serum electrolytes. c. Administer intravenous furosemide (Lasix). d. Ask the client about current medications.

ANS: A Assessment of respiratory and oxygenation status is the priority nursing intervention for the prevention of complications. Monitoring electrolytes, administering diuretics, and asking about current medications are important but do not take priority over assessing respiratory status. DIF: Applying/Application REF: 687 KEY: Heart failure| respiratory distress/failure| assessment/diagnostic examination MSC: Integrated Process: Nursing Process: Implementation NOT: Client Needs Category: Safe and Effective Care Environment: Management of Care

15. The nurse is careful to monitor a patient's cardiac output. Which goal is the nurse trying to achieve? a. To determine peripheral extremity circulation b. To determine oxygenation requirements c. To determine cardiac dysrhythmias d. To determine ventilation status

ANS: A Cardiac output indicates how much blood is being circulated systemically throughout the body to the periphery. The amount of blood ejected from the left ventricle each minute is the cardiac output. Oxygen status would be determined by pulse oximetry and the presence of cyanosis. Cardiac dysrhythmias are an electrical impulse monitored through ECG results. Ventilation status is measured by respiratory rate, pulse oximetry, and capnography. Capnography provides instant information about the patient's ventilation. Ventilation status does not depend solely on cardiac output.

40. A patient with chronic obstructive pulmonary disease (COPD) asks the nurse why clubbing occurs. Which response by the nurse is most therapeutic? a. "Your disease doesn't send enough oxygen to your fingers." b. "Your disease affects both your lungs and your heart, and not enough blood is being pumped." c. "Your disease will be helped if you pursed-lip breathe." d. "Your disease often makes patients lose mental status."

ANS: A Clubbing of the nail bed can occur with COPD and other diseases that cause prolonged oxygen deficiency or chronic hypoxemia. Pursed-lipped breathing helps the alveoli stay open but is not the cause of clubbing. Loss of mental status is not a normal finding with COPD and will not result in clubbing. Low oxygen and not low circulating blood volume is the problem in COPD that results in clubbing.

5. A nurse cares for a client with right-sided heart failure. The client asks, "Why do I need to weigh myself every day?" How should the nurse respond? a. "Weight is the best indication that you are gaining or losing fluid." b. "Daily weights will help us make sure that you're eating properly." c. "The hospital requires that all inpatients be weighed daily." d. "You need to lose weight to decrease the incidence of heart failure."

ANS: A Daily weights are needed to document fluid retention or fluid loss. One liter of fluid equals 2.2 pounds. The other responses do not address the importance of monitoring fluid retention or loss. DIF: Remembering/Knowledge REF: 683 KEY: Heart failure| patient education MSC: Integrated Process: Teaching/Learning NOT: Client Needs Category: Physiological Integrity: Physiological Adaptation

21. A nurse cares for an older adult client with heart failure. The client states, "I don't know what to do. I don't want to be a burden to my daughter, but I can't do it alone. Maybe I should die." How should the nurse respond? a. "Would you like to talk more about this?" b. "You are lucky to have such a devoted daughter." c. "It is normal to feel as though you are a burden." d. "Would you like to meet with the chaplain?"

ANS: A Depression can occur in clients with heart failure, especially older adults. Having the client talk about his or her feelings will help the nurse focus on the actual problem. Open-ended statements allow the client to respond safely and honestly. The other options minimize the client's concerns and do not allow the nurse to obtain more information to provide client-centered care. DIF: Applying/Application REF: 683 KEY: Heart failure| support| psychosocial response MSC: IntegratedProcess:Caring NOT: Client Needs Category: Psychosocial Integrity

11. The emergency department (ED) manager is reviewing client charts to determine how well the staff perform when treating clients with community-acquired pneumonia. What outcome demonstrates that goals for this client type have been met? a. Antibiotics started before admission. b. Blood cultures obtained within 20 minutes. c. Chest x-ray obtained within 30 minutes. d. Pulse oximetry obtained on all clients.

ANS: A Goals for treatment of community-acquired pneumonia include initiating antibiotics prior to inclient admission or within 6 hours of presentation to the ED. Timely collection of blood cultures, chest x-ray, and pulse oximetry are important as well but do not coincide with established goals.

7. The patient is breathing normally. Which process does the nurse consider is working properly when the patient inspires? a. Stimulation of chemical receptors in the aorta b. Reduction of arterial oxygen saturation levels c. Requirement of elastic recoil lung properties d. Enhancement of accessory muscle usage

ANS: A Inspiration is an active process, stimulated by chemical receptors in the aorta. Reduced arterial oxygen saturation levels indicate hypoxemia, an abnormal finding. Expiration is a passive process that depends on the elastic recoil properties of the lungs, requiring little or no muscle work. Prolonged use of the accessory muscles does not promote effective ventilation and causes fatigue.

29. A nurse is caring for a patient with chronic obstructive pulmonary disease (COPD) who is receiving 2 L/min of oxygen. Which oxygen delivery device is most appropriate for the nurse to administer the oxygen? a. Nasal cannula b. Simple face mask c. Non-rebreather mask d. Partial non-rebreather mask

ANS: A Nasal cannulas deliver oxygen from 1 to 6 L/min. All other devices (simple face mask, non-rebreather mask, and partial non-rebreather mask) are intended for flow rates greater than 6 L/min.

11. A client is receiving oxygen at 4 liters per nasal cannula. What comfort measure may the nurse delegate to unlicensed assistive personnel (UAP)? a. Apply water-soluble ointment to nares and lips. b. Periodically turn the oxygen down or off. c. Remove the tubing from the client's nose. d. Turn the client every 2 hours or as needed.

ANS: A Oxygen can be drying, so the UAP can apply water-soluble lubricant to the client's lips and nares. The UAP should not adjust the oxygen flow rate or remove the tubing. Turning the client is not related to comfort measures for oxygen.

32. The nurse is caring for a patient with fluid volume overload. Which physiological effect does the nurse most likely expect? a. Increased preload b. Increased heart rate c. Decreased afterload d. Decreased tissue perfusion

ANS: A Preload refers to the amount of blood in the left ventricle at the end of diastole; an increase in circulating volume would increase the preload of the heart. Afterload refers to resistance; increased pressure would lead to increased resistance, and afterload would increase. A decrease in tissue perfusion would be seen with hypovolemia. A decrease in fluid volume would cause an increase in heart rate as the body is attempting to increase cardiac output.

7. A student is practicing suctioning a tracheostomy in the skills laboratory. What action by the student demonstrates that more teaching is needed? a. Applying suction while inserting the catheter b. Preoxygenating the client prior to suctioning c. Suctioning for a total of three times if needed d. Suctioning for only 10 to 15 seconds each time

ANS: A Suction should only be applied while withdrawing the catheter. The other actions are appropriate.

6. A client admitted for pneumonia has been tachypneic for several days. When the nurse starts an IV to give fluids, the client questions this action, saying "I have been drinking tons of water. How am I dehydrated?" What response by the nurse is best? a. "Breathing so quickly can be dehydrating." b. "Everyone with pneumonia is dehydrated." c. "This is really just to administer your antibiotics." d. "Why do you think you are so dehydrated?"

ANS: A Tachypnea and mouth breathing (from increased work of breathing), both seen in pneumonia, increase insensible water loss and can lead to a degree of dehydration. The other options do not give the client useful information that addresses this specific concern.

20. A nurse is reviewing the electrocardiogram (ECG) results. Which portion of the conduction system does the nurse consider when evaluating the P wave? a. SA node b. AV node c. Bundle of His d. Purkinje fibers

ANS: A The P wave represents the electrical conduction through both atria; the SA node initiates electrical conduction through the atria. The AV node conducts down through the bundle of His and the Purkinje fibers to cause ventricular contraction.

3. A nurse explains the function of the alveoli to a patient with respiratory problems. Which information about the alveoli's function will the nurse share with the patient? a. Carries out gas exchange b. Regulates tidal volume c. Produces hemoglobin d. Stores oxygen

ANS: A The alveolus is a capillary membrane that allows gas exchange of oxygen and carbon dioxide during respiration. The alveoli do not store oxygen, regulate tidal volume, or produce hemoglobin.

19. A nurse assesses a client who has aortic regurgitation. In which location in the illustration shown below would the nurse auscultate to best hear a cardiac murmur related to aortic regurgitation? a. Location A b. Location B c. Location C d. Location D

ANS: A The aortic valve is auscultated in the second intercostal space just to the right of the sternum. The pulmonic valve would be heard in location B located in the second intercostal space just left of the sternum. The mitral valve would be heard in location D located in the fifth intercostal space at the apex of the heart. The tricuspid valve would be heard in location C located in the fifth intercostal space at the lower left of the sternal border.

44. Which coughing technique will the nurse use to help a patient clear central airways? a. Huff b. Quad c. Cascade d. Incentive spirometry

ANS: A The huff cough stimulates a natural cough reflex and is generally effective only for clearing central airways. While exhaling, the patient opens the glottis by saying the word huff. The quad cough technique is for patients without abdominal muscle control such as those with spinal cord injuries. While the patient breathes out with a maximal expiratory effort, the patient or nurse pushes inward and upward on the abdominal muscles toward the diaphragm, causing the cough. With the cascade cough the patient takes a slow, deep breath and holds it for 2 seconds while contracting expiratory muscles. Then he or she opens the mouth and performs a series of coughs throughout exhalation, thereby coughing at progressively lowered lung volumes. This technique promotes airway clearance and a patent airway in patients with large volumes of sputum. Incentive spirometry encourages voluntary deep breathing by providing visual feedback to patients about inspiratory volume. It promotes deep breathing and prevents or treats atelectasis in the postoperative patient.

16. A nurse is caring for a group of patients. Which patient should the nurse see first? a. A patient with hypercapnia wearing an oxygen mask b. A patient with a chest tube ambulating with the chest tube unclamped c. A patient with thick secretions being tracheal suctioned first and then orally d. A patient with a new tracheostomy and tracheostomy obturator at bedside

ANS: A The mask is contraindicated for patients with carbon dioxide retention (hypercapnia) because retention can be worsened; the nurse must see this patient first to correct the problem. All the rest are using correct procedures and do not need to be seen first. A chest tube should not be clamped when ambulating. Clamping a chest tube is contraindicated when ambulating or transporting a patient. Clamping can result in a tension pneumothorax. Use nasotracheal suctioning before pharyngeal suctioning whenever possible. The mouth and pharynx contain more bacteria than the trachea. Keep tracheostomy obturator at bedside with a fresh (new) tracheostomy to facilitate reinsertion of the outer cannula if dislodged.

12. A nurse assesses a client who is recovering from a myocardial infarction. The client's blood pressure is 140/88 mm Hg. What action would the nurse take first? a. Compare the results with previous blood pressure readings. b. Increase the intravenous fluid rate because these readings are low. c. Immediately notify the primary health care provider of the elevated blood pressure. d. Document the finding in the client's chart as the only action.

ANS: A The most recent range for normal blood pressure is less than 140 mm Hg systolic and less than 90mm Hg diastolic. This client's blood pressure is at the upper range of acceptable, so the nurse would compare the client's current reading with those previously recorded before doing anything else. The reading is not low, so the nurse would not increase IV fluids, nor would the nurse necessarily notify the primary health care provider. Documentation is important, but the nurse first checks previous readings.

15. A client has been hospitalized with tuberculosis (TB). The client's spouse is fearful of entering the room where the client is in isolation and refuses to visit. What action by the nurse is best? a. Ask the spouse to explain the fear of visiting in further detail. b. Inform the spouse that the precautions are meant to keep other clients safe. c. Show the spouse how to follow the Isolation Precautions to avoid illness. d. Tell the spouse that he or she has already been exposed, so it's safe to visit.

ANS: A The nurse needs to obtain further information about the spouse's specific fears so they can be addressed. This will decrease stress and permit visitation, which will be beneficial for both client and spouse. Precautions for TB prevent transmission to all who come into contact with the client. Explaining Isolation Precautions and what to do when entering the room will be helpful, but this is too narrow in scope to be the best answer. Telling the spouse that it's safe to visit is demeaning of the spouse's feelings.

4. While assessing a client on a cardiac unit, a nurse identifies the presence of an S3 gallop. Which action should the nurse take next? a. Assess for symptoms of left-sided heart failure. b. Document this as a normal finding. c. Call the health care provider immediately. d. Transfer the client to the intensive care unit.

ANS: A The presence of an S3 gallop is an early diastolic filling sound indicative of increasing left ventricular pressure and left ventricular failure. The other actions are not warranted. DIF: Remembering/Knowledge REF: 683 KEY: Heart failure| assessment/diagnostic examination MSC: IntegratedProcess:NursingProcess:Assessment NOT: Client Needs Category: Physiological Integrity: Reduction of Risk Potential

5. An unlicensed assistive personnel (UAP) was feeding a client with a tracheostomy. Later that evening, the UAP reports that the client had a coughing spell during the meal. What action by the nurse takes priority? a. Assess the client's lung sounds. b. Assign a different UAP to the client. c. Report the UAP to the manager. d. Request thicker liquids for meals.

ANS: A The priority is to check the client's oxygenation because he or she may have aspirated. Once the client has been assessed, the nurse can consult with the registered dietitian about appropriately thickened liquids. The UAP should have reported the incident immediately, but addressing that issue is not the immediate priority.

8. A client has been diagnosed with tuberculosis (TB). What action by the nurse takes highest priority? a. Educating the client on adherence to the treatment regimen b. Encouraging the client to eat a well-balanced diet c. Informing the client about follow-up sputum cultures d. Teaching the client ways to balance rest with activity

ANS: A The treatment regimen for TB often ranges from 26 weeks, but can be up to 2 years, making adherence problematic for many people. The nurse would stress the absolute importance of following the treatment plan for the entire duration of prescribed therapy. The other options are appropriate topics to educate this client on but do not take priority.

3. A client has a tracheostomy that is 3 days old. Upon assessment, the nurse notes the client's face is puffy and the eyelids are swollen. What action by the nurse takes priority? a. Assess the client's oxygen saturation. b. Notify the Rapid Response Team. c. Oxygenate the client with a bag-valve-mask. d. Palpate the skin of the upper chest.

ANS: A This client may have subcutaneous emphysema, which is air that leaks into the tissues surrounding the tracheostomy. The nurse should first assess the client's oxygen saturation and other indicators of oxygenation. If the client is stable, the nurse can palpate the skin of the upper chest to feel for the air. If the client is unstable, the nurse calls the Rapid Response Team. Using a bag-valve-mask device may or may not be appropriate for the unstable client.

36. The nurse is creating a plan of care for an obese patient who is suffering from fatigue related to ineffective breathing. Which intervention best addresses a short-term goal the patient could achieve? a. Sleeping on two to three pillows at night b. Limiting the diet to 1500 calories a day c. Running 30 minutes every morning d. Stopping smoking immediately

ANS: A To achieve a short-term goal, the nurse should plan a lifestyle change that the patient can make immediately that will have a quick effect. Sleeping on several pillows at night will immediately relieve orthopnea and open the patient's airway, thereby reducing fatigue. Running 30 minutes a day will improve cardiopulmonary health, but a patient needs to build up exercise tolerance. Smoking cessation is another process that many people have difficulty doing immediately. A more realistic short-term goal would be to gradually reduce the number of cigarettes smoked. Limiting caloric intake can help a patient lose weight, but this is a gradual process and is not reasonable for a short-term goal.

33. A nurse is caring for a patient with continuous cardiac monitoring for heart dysrhythmias. Which rhythm will cause the nurse to intervene immediately? a. Ventricular tachycardia b. Atrial fibrillation c. Sinus rhythm d. Paroxysmal supraventricular tachycardia

ANS: A Ventricular tachycardia and ventricular fibrillation are life-threatening rhythms that require immediate intervention. Ventricular tachycardia is a life-threatening dysrhythmia because of the decreased cardiac output and the potential to deteriorate into ventricular fibrillation or sudden cardiac death. Atrial fibrillation is a common dysrhythmia in older adults and is not as serious as ventricular tachycardia. Sinus rhythm is normal. Paroxysmal supraventricular tachycardia is a sudden, rapid onset of tachycardia originating above the AV node. It often begins and ends spontaneously.

1. A nurse assesses a client who had a myocardial infarction and has a blood pressure of 88/58 mm Hg. Which additional assessment finding would the nurse expect? a. Heart rate of 120 beats/min b. Cool, clammy skin c. Oxygen saturation of 90% d. Respiratory rate of 8 breaths/min

ANS: A When a client experiences hypotension, baroreceptors in the aortic arch sense a pressure decrease in the vessels. The parasympathetic system responds by lessening the inhibitory effect on the sinoatrial node. This results in an increase in heart rate and respiratory rate. This tachycardia is an early response and is seen even when blood pressure is not critically low. An increased heart rate and respiratory rate will compensate for the low blood pressure and maintain oxygen saturation and perfusion. The client may not be able to compensate for long and decreased oxygenation and cool, clammy skin will occur later.

35. A nurse is teaching about risk factors for cardiopulmonary disease. Which risk factor should the nurse describe as modifiable? a. Stress b. Allergies c. Family history d. Gender

ANS: A Young and middle-age adults are exposed to multiple cardiopulmonary risk factors: an unhealthy diet, lack of exercise, stress, over-the-counter and prescription drugs not used as intended, illegal substances, and smoking. Reducing these modifiable factors decreases a patient's risk for cardiac or pulmonary diseases. A nonmodifiable risk factor is family history; determine familial risk factors such as a family history of lung cancer or cardiovascular disease. Other nonmodifiable risk factors include allergies and gender.

The nurse working with oncology clients understands that interacting factors affect cancer development. Which factors does this include? (Select all that apply.) a. Exposure to carcinogens b. Genetic predisposition c. Immune function d. Normal doubling time e. State of euploidy

ANS: A, B, C The three interacting factors needed for cancer development are exposure to carcinogens, genetic predisposition, and immune function.

1. A home health nurse is visiting a new client who uses oxygen in the home. For which factors does the nurse assess when determining if the client is using the oxygen safely? (Select all that apply.) a. The client does not allow smoking in the house. b. Electrical cords are in good working order. c. Flammable liquids are stored in the garage. d. Household light bulbs are the fluorescent type. e. The client does not have pets inside the home.

ANS: A, B, C Oxygen is an accelerant, which means it enhances combustion, so precautions are needed whenever using it. The nurse should assess if the client allows smoking near the oxygen, whether electrical cords are in good shape or are frayed, and if flammable liquids are stored (and used) in the garage away from the oxygen. Light bulbs and pets are not related to oxygen safety.

A nurse is providing community education on the seven warning signs of cancer. Which signs are included? (Select all that apply.) a. A sore that does not heal b. Changes in menstrual patterns c. Indigestion or trouble swallowing d. Near-daily abdominal pain e. Obvious change in a mole f. Frequent indigestion

ANS: A, B, C, E, F The seven warning signs for cancer can be remembered with the acronym CAUTION: changes in bowel or bladder habits, a sore that does not heal, unusual bleeding or discharge, thickening or lump in the breast or elsewhere, indigestion or difficulty swallowing, obvious change in a wart or mole, and nagging cough or hoarseness. Abdominal pain is not a warning sign.

8. A nurse assesses a client who is recovering from a heart transplant. Which assessment findings should alert the nurse to the possibility of heart transplant rejection? (Select all that apply.) a. Shortness of breath b. Abdominal bloating c. New-onset bradycardia d. Increased ejection fraction e. Hypertension f. Fatigue

ANS: A, B, C, F Clinical manifestations of heart transplant rejection include shortness of breath, fatigue, fluid gain, abdominal bloating, new-onset bradycardia, hypotension, atrial fibrillation or flutter, decreased activity tolerance, and decreased ejection fraction. DIF: Remembering/Knowledge REF: 703 KEY: Transplant| heart failure MSC: IntegratedProcess:NursingProcess:Assessment NOT: Client Needs Category: Physiological Integrity: Reduction of Risk Potential

3. A nurse assesses clients on a cardiac unit. Which clients should the nurse identify as at greatest risk for the development of acute pericarditis? (Select all that apply.) a. A 36-year-old woman with systemic lupus erythematosus (SLE) b. A 42-year-old man recovering from coronary artery bypass graft surgery c. A 59-year-old woman recovering from a hysterectomy d. An 80-year-old man with a bacterial infection of the respiratory tract e. An 88-year-old woman with a stage III sacral ulcer

ANS: A, B, D Acute pericarditis is most commonly associated acute exacerbations of systemic connective tissue disease, including SLE; with Dressler's syndrome, or inflammation of the cardiac sac after cardiac surgery or a myocardial infarction; and with infective organisms, including bacterial, viral, and fungal infections. Abdominal and reproductive surgeries and pressure ulcers do not increase clients' risk for acute pericarditis. DIF: Applying/Application REF: 699 KEY: Inflammatory response| health screening MSC: IntegratedProcess:NursingProcess:Assessment NOT: Client Needs Category: Safe and Effective Care Environment: Management of Care

7. A nurse prepares to discharge a client who has heart failure. Which questions should the nurse ask to ensure this client's safety prior to discharging home? (Select all that apply.) a. "Are your bedroom and bathroom on the first floor?" b. "What social support do you have at home?" c. "Will you be able to afford your oxygen therapy?" d. "What spiritual beliefs may impact your recovery?" e. "Are you able to accurately weigh yourself at home?"

ANS: A, B, D To ensure safety upon discharge, the nurse should assess for structural barriers to functional ability, such as stairs. The nurse should also assess the client's available social support, which may include family, friends, and home health services. The client's ability to adhere to medication and treatments, including daily weights, should also be reviewed. The other questions do not address the client's safety upon discharge. DIF: Applying/Application REF: 689 KEY: Heart failure| discharge| safety MSC: Integrated Process: Nursing Process: Analysis NOT: Client Needs Category: Safe and Effective Care Environment: Safety and Infection Control

4. A nurse is planning discharge teaching on tracheostomy care for an older client. What factors does the nurse need to assess before teaching this particular client? (Select all that apply.) a. Cognition b. Dexterity c. Hydration d. Range of motion e. Vision

ANS: A, B, D, E The older adult is at risk for having impairments in cognition, dexterity, range of motion, and vision that could limit the ability to perform tracheostomy care and should be assessed. Hydration is not directly related to the ability to perform self-care.

MULTIPLE RESPONSE 1. A nurse is assessing a client with left-sided heart failure. For which clinical manifestations should the nurse assess? (Select all that apply.) a. Pulmonary crackles b. Confusion, restlessness c. Pulmonary hypertension d. Dependent edema e. Cough that worsens at night

ANS: A, B, E Left-sided heart failure occurs with a decrease in contractility of the heart or an increase in afterload. Most of the signs will be noted in the respiratory system. Right-sided heart failure occurs with problems from the pulmonary vasculature onward including pulmonary hypertension. Signs will be noted before the right atrium or ventricle including dependent edema. DIF: Remembering/Knowledge REF: 682 KEY: Heart failure| assessment/diagnostic examination MSC: IntegratedProcess:NursingProcess:Assessment NOT: Client Needs Category: Physiological Integrity: Physiological Adaptation

2. A nurse evaluates laboratory results for a client with heart failure. Which results should the nurse expect? (Select all that apply.) a. Hematocrit: 32.8% b. Serum sodium: 130 mEq/L c. Serum potassium: 4.0 mEq/L d. Serum creatinine: 1.0 mg/dL e. Proteinuria f. Microalbuminuria

ANS: A, B, E, F A hematocrit of 32.8% is low (should be 42.6%), indicating a dilutional ratio of red blood cells to fluid. A serum sodium of 130 mEq/L is low because of hemodilution. Microalbuminuria and proteinuria are present, indicating a decrease in renal filtration. These are early warning signs of decreased compliance of the heart. The potassium level is on the high side of normal and the serum creatinine level is normal. DIF: Applying/Application REF: 683 KEY: Heart failure| assessment/diagnostic examination MSC: Integrated Process: Nursing Process: Evaluation NOT: Client Needs Category: Physiological Integrity: Reduction of Risk Potential

5. A nurse collaborates with an unlicensed assistive personnel (UAP) to provide care for a client with congestive heart failure. Which instructions should the nurse provide to the UAP when delegating care for this client? (Select all that apply.) a. "Reposition the client every 2 hours." b. "Teach the client to perform deep-breathing exercises." c. "Accurately record intake and output." d. "Use the same scale to weigh the client each morning." e. "Place the client on oxygen if the client becomes short of breath."

ANS: A, C, D The UAP should reposition the client every 2 hours to improve oxygenation and prevent atelectasis. The UAP can also accurately record intake and output, and use the same scale to weigh the client each morning before breakfast. UAPs are not qualified to teach clients or assess the need for and provide oxygen therapy. DIF: Applying/Application REF: 684 KEY: Heart failure| delegation| interdisciplinary team| unlicensed assistive personnel (UAP) MSC: Integrated Process: Communication and Documentation NOT: Client Needs Category: Safe and Effective Care Environment: Management of Care

2. A nurse is caring for a client who has a tracheostomy tube. What actions may the nurse delegate to unlicensed assistive personnel (UAP)? (Select all that apply.) a. Applying water-soluble lip balm to the client's lips b. Ensuring the humidification provided is adequate c. Performing oral care with alcohol-based mouthwash d. Reminding the client to cough and deep breathe often e. Suctioning excess secretions through the tracheostomy

ANS: A, D The UAP can perform hygiene measures such as applying lip balm and reinforce teaching such as reminding the client to perform coughing and deep-breathing exercises. Oral care can be accomplished with normal saline, not products that dry the mouth. Ensuring the humidity is adequate and suctioning through the tracheostomy are nursing functions.

4. After teaching a client with congestive heart failure (CHF), the nurse assesses the client's understanding. Which client statements indicate a correct understanding of the teaching related to nutritional intake? (Select all that apply.) a. "I'll read the nutritional labels on food items for salt content." b. "I will drink at least 3 liters of water each day." c. "Using salt in moderation will reduce the workload of my heart." d. "I will eat oatmeal for breakfast instead of ham and eggs." e. "Substituting fresh vegetables for canned ones will lower my salt intake."

ANS: A, D, E Nutritional therapy for a client with CHF is focused on decreasing sodium and water retention to decrease the workload of the heart. The client should be taught to read nutritional labels on all food items, omit table salt and foods high in sodium (e.g., ham and canned foods), and limit water intake to a normal 2 L/day. DIF: Applying/Application REF: 682 KEY: Heart failure| patient education MSC: Integrated Process: Teaching/Learning NOT: Client Needs Category: Physiological Integrity: Basic Care and Comfort

3. A client is being discharged home after having a tracheostomy placed. What suggestions does the nurse offer to help the client maintain self-esteem? (Select all that apply.) a. Create a communication system. b. Don't go out in public alone. c. Find hobbies to enjoy at home. d. Try loose-fitting shirts with collars. e. Wear fashionable scarves.

ANS: A, D, E The client with a tracheostomy may be shy and hesitant to go out in public. The client should have a sound communication method to ease frustration. The nurse can also suggest ways of enhancing appearance so the client is willing to leave the house. These can include wearing scarves and loose-fitting shirts to hide the stoma. Keeping the client homebound is not good advice.

The nurse learning about cancer development remembers characteristics of normal cells. Which characteristics does this include? (Select all that apply.) a. Differentiated function b. Large nucleus-to-cytoplasm ratio c. Loose adherence d. Nonmigratory e. Specific morphology f. Orderly and specific growth

ANS: A, D, E, F Normal cells have the characteristics of differentiated function, nonmigratory, specific morphology, a smaller nucleus-to-cytoplasm ratio, tight adherence, and orderly and well-regulated growth.

50. A nurse is teaching a community health promotion class and discusses the flu vaccine. Which information will the nurse include in the teaching session? (Select all that apply.) a. It is given yearly. b. It is given in a series of four doses. c. It is safe for children allergic to eggs. d. It is safe for adults with acute febrile illnesses. e. The nasal spray is given to people over 50. f. The inactivated flu vaccine is given to people over 50.

ANS: A, F Annual (yearly) flu vaccines are recommended for all people 6 months and older. The inactivated flu vaccine should be given to these individuals with chronic health problems and those 50 and older. People with a known hypersensitivity to eggs or other components of the vaccine should consult their health care provider before being vaccinated. There is a flu vaccine made without egg proteins that is approved for adults 18 years of age and older. Adults with an acute febrile illness should schedule the vaccination after they have recovered. The live, attenuated nasal spray vaccine is given to people from 2 through 49 years of age if they are not pregnant or do not have certain long-term health problems such as asthma; heart, lung, or kidney disease; diabetes; or anemia.

1. A nurse is caring for a client with a history of renal insufficiency who is scheduled for a cardiac catheterization. What actions would the nurse take prior to the catheterization? (Select all that apply.) a. Assess for allergies to iodine. b. Administer intravenous fluids. c. Assess blood urea nitrogen (BUN) and creatinine results. d. Insert a Foley catheter. e. Administer a prophylactic antibiotic. f. Insert a central venous catheter.

ANS: A,B,C If the client has kidney disease, fluids may be given 12 to 24 hours before the procedure for renal protection. Hydration would continue after the procedure. The client would be assessed for allergies to iodine, including shellfish; the contrast medium used during the catheterization contains iodine. Baseline renal labs would be assessed. A Foley catheter and central venous catheter are not required for the procedure and would only increase the client's risk for infection. Prophylactic antibiotics are not administered prior to a cardiac catheterization.

3. A client in the emergency department is taking rifampin for tuberculosis. The client reports yellowing of the sclera and skin and bleeding after minor trauma. What laboratory results correlate to this condition? (Select all that apply.) 4. A client has been diagnosed with an empyema. What interventions would the nurse anticipate providing to this client? (Select all that apply.) a. Assisting with chest tube insertion b. Facilitating pleural fluid sampling c. Performing frequent respiratory assessment d. Providing antipyretics as needed e. Suctioning deeply every 4 hours

ANS: A,B,C,D The client with an empyema is often treated with chest tube insertion, which facilitates obtaining samples of the pleural fluid for analysis and re-expands the lungs. The nurse would perform frequent respiratory system assessments. Antipyretic medications are also used. Suction is only used when needed and is not done deeply to prevent tissue injury.

6. A client is taking ethambutol for tuberculosis. What instructions does the nurse provide the client regarding this drug? (Select all that apply.) a. Contact the primary health care provider if preexisting gout becomes worse. b. Report any changes in vision immediately to the health care provider. c. Avoid drinking alcoholic beverages due to the chance of liver damage. d. Do not take antacids or eat within 2 hours after taking this medication. e. You will take this medication along with some others for 8 weeks. f. Take this medicine with a full glass of water.

ANS: A,B,E,F The nurse would teach the client that preexisting gout may get worse and the client should report this as medications for gout may need to be adjusted. The nurse would also inform the client about the multi-drug routine used for TB. Optic neuritis can occur with this drug so the client needs to report visual changes right away. The medication should be taken with a full glass of water. Drinking while taking ethambutol causes severe nausea and vomiting. Avoiding antacids and food (within 2 hours) is a precaution with isoniazid.

1. A nurse is providing pneumonia vaccinations in a community setting. Due to limited finances, the event organizers must limit giving the vaccination to priority groups. What clients would be considered a priority when administering the pneumonia vaccination? (Select all that apply.) a. A 22-year-old client with asthma b. Client who had a cholecystectomy last year c. Client with well-controlled diabetes d. Healthy 72-year-old client e. Client who is taking medication for hypertension

ANS: A,C,D,E Clients over 65 years of age and any client (no matter what age) with a chronic health condition would be considered a priority for a pneumonia vaccination. Having a cholecystectomy a year ago does not qualify as a chronic health condition.

4. A nurse reviews a client's laboratory results. Which findings would alert the nurse to the possibility of atherosclerosis? (Select all that apply.) a. Total cholesterol: 280 mg/dL (7.3 mmol/L) b. High-density lipoprotein cholesterol: 50 mg/dL (1.3 mmol/L) c. Triglycerides: 200 mg/dL (2.3 mmol/L) d. Serum albumin: 4 g/dL (5.8 mcmol/L) e. Low-density lipoprotein cholesterol: 160 mg/dL (4.1 mmol/L)*

ANS: A,C,E A lipid panel is often used to screen for cardiovascular risk. Total cholesterol, triglycerides, and low-density lipoprotein cholesterol levels are all high, indicating higher risk for cardiovascular disease. High-density lipoprotein cholesterol is within the normal range for both males and females. Serum albumin is not assessed for atherosclerosis.

6. A nurse cares for a client who is recovering from a right-sided heart catheterization. For which complications of this procedure would the nurse assess? (Select all that apply.) a. Thrombophlebitis b. Stroke c. Pulmonary embolism d. Myocardial infarction e. Cardiac tamponade f. Dysrhythmias

ANS: A,C,E Complications from a right-sided heart catheterization include thrombophlebitis, pulmonary embolism, and vagal response. Cardiac tamponade is a risk of both right- and left-sided heart catheterizations. Stroke, myocardial infarction, and dysrhythmias are complications of left-sided heart catheterizations.

7. The nurse is learning about endemic pulmonary diseases. Which diseases are matched with correct information? (Select all that apply.) a. Hanta virus: found in urine, droppings, and saliva of infected rodents. b. Aspergillosis: requires a prolonged course of antibiotics. c. Histoplasmosis: sources include soil containing bird and bat droppings. d. Blastomycosis: requires strict adherence to multi-antibiotic regimen. e. Cryptococcosis: has been eradicated due to strategic deforestation. f. Coccidioidomycosis: found in the southwest and far west of the United States.

ANS: A,C,F Hanta virus is often seen in the southwest United States and is found in the urine, droppings, and saliva of infected rodents. Histoplasmosis is found in soil containing bird and bat droppings and on surfaces covered with bird droppings. Apergillosis is a common mold found both indoors and outdoors and is treated with a long course of antifungal drugs. Blastomycosis is a fungal disease requiring a prolonged course of antifungal medications. Cryptococcosis is a fungus found on trees and in the soil beneath trees, but has not been eradicated with strategic deforestation. Coccidioidomycosis is found in the southwest and far west of the United States, plus Mexico, and Central and South America.

The nurse learning about cellular regulation understands that which process occurs during the S phase of the cell cycle? a. Actual division (mitosis) b. Doubling of DNA c. Growing extra membrane d. No reproductive activity

ANS: B During the S phase, the cell must double its DNA content through DNA synthesis. Actual division, or mitosis, occurs during the M phase. Growing extra membrane occurs in the G1 phase. During the G0 phase, the cell is working but is not involved in any reproductive activity.

A nurse learns that which of the following is the single biggest risk factor for developing cancer? a. Exposure to tobacco b. Advancing age c. Occupational chemicals d. Oncovirus infection

ANS: B The single biggest risk factor for developing cancer is advancing age. As one ages, immunity decreases and exposures increase. Tobacco use is the single most preventable cause of cancer. Exposure to chemicals and oncoviruses cause fewer cancers.

22. A nurse teaches a client with heart failure about energy conservation. Which statement should the nurse include in this client's teaching? a. "Walk until you become short of breath, and then walk back home." b. "Gather everything you need for a chore before you begin." c. "Pull rather than push or carry items heavier than 5 pounds." d. "Take a walk after dinner every day to build up your strength."

ANS: B A client who has heart failure should be taught to conserve energy. Gathering all supplies needed for a chore at one time decreases the amount of energy needed. The client should not walk until becoming short of breath because he or she may not make it back home. Pushing a cart takes less energy than pulling or lifting. Although walking after dinner may help the client, the nurse should teach the client to complete activities when he or she has the most energy. This is usually in the morning. DIF: Applying/Application REF: 696 KEY: Heart failure| functional ability| patient education MSC: IntegratedProcess:Teaching/Learning NOT: Client Needs Category: Health Promotion and Maintenance

31. The nurse determines that an older-adult patient is at risk for infection due to decreased immunity. Which plan of care best addresses the prevention of infection for the patient? a. Inform the patient of the importance of finishing the entire dose of antibiotics. b. Encourage the patient to stay up-to-date on all vaccinations. c. Schedule patient to get annual tuberculosis skin testing. d. Create an exercise routine to run 45 minutes every day.

ANS: B A nursing care plan for preventative health measures should be reasonable and feasible. Keeping up-to-date on vaccinations is important because vaccine reduces the severity of illnesses and serious complications. Determine if and when the patient has had a pneumococcal or influenza (flu) vaccine. This is especially important when assessing older adults because of their increased risk for respiratory disease. Although it is important to finish the full course of antibiotics, it is not a preventative health measure. Scheduling annual tuberculosis skin tests does not address prevention and is an unreliable indictor of tuberculosis in older patients. The exercise routine should be reasonable to increase compliance; exercise is recommended only 3 to 4 times a week for 30 to 60 minutes, and walking, rather than running, is an efficient method.

7. After administering newly prescribed captopril (Capoten) to a client with heart failure, the nurse implements interventions to decrease complications. Which priority intervention should the nurse implement for this client? a. Provide food to decrease nausea and aid in absorption. b. Instruct the client to ask for assistance when rising from bed. c. Collaborate with unlicensed assistive personnel to bathe the client. d. Monitor potassium levels and check for symptoms of hypokalemia.

ANS: B Administration of the first dose of angiotensin-converting enzyme (ACE) inhibitors is often associated with hypotension, usually termed first-dose effect. The nurse should instruct the client to seek assistance before arising from bed to prevent injury from postural hypotension. ACE inhibitors do not need to be taken with food. Collaboration with unlicensed assistive personnel to provide hygiene is not a priority. The client should be encouraged to complete activities of daily living as independently as possible. The nurse should monitor for hyperkalemia, not hypokalemia, especially if the client has renal insufficiency secondary to heart failure. DIF: Applying/Application REF: 685 KEY: Heart failure| angiotensin-converting enzyme (ACE) inhibitor| medication| patient education MSC: Integrated Process: Nursing Process: Implementation NOT: Client Needs Category: Physiological Integrity: Pharmacological and Parenteral Therapies

30. The nurse needs to closely monitor the oxygen status of an older-adult patient undergoing anesthesia because of which age-related change? a. Thinner heart valves cause lipid accumulation and fibrosis. b. Diminished respiratory muscle strength may cause poor chest expansion. c. Alterations in mental status prevent patients' awareness of ineffective breathing. d. An increased number of pacemaker cells make proper anesthesia induction more difficult.

ANS: B Age-related changes in the thorax that occur from ossification of costal cartilage, decreased space between vertebrae, and diminished respiratory muscle strength lead to problems with chest expansion and oxygenation,whereby the patient will have difficulty excreting anesthesia gas. The nurse needs to monitor the patient's oxygen status carefully to make sure the patient does not retain too much of the drug. Older adults experience alterations in cardiac function as a result of calcification of the conduction pathways, thicker and stiffer heart valves caused by lipid accumulation and fibrosis, and a decrease in the number of pacemaker cells in the SA node. Altered mental status is not a normal age-related change; it indicates possible cardiac and/or respiratory problems.

3. Which teaching point is most important for the client with a peritonsillar abscess? a. Gargle with warm salt water. b. Take all antibiotics as directed. c. Let us know if you want liquid medications. d. Wash hands frequently.

ANS: B Any client on antibiotics must be instructed to complete the entire course of antibiotics. Not completing them can lead to complications or drug-resistant strains of bacteria. The other instructions are appropriate, just not the most important.

21. A nurse teaches a patient about atelectasis. Which statement by the patient indicates an understanding of atelectasis? a. "Atelectasis affects only those with chronic conditions such as emphysema." b. "It is important to do breathing exercises every hour to prevent atelectasis." c. "If I develop atelectasis, I will need a chest tube to drain excess fluid." d. "Hyperventilation will open up my alveoli, preventing atelectasis."

ANS: B Atelectasis develops when alveoli do not expand. Breathing exercises, especially deep breathing and incentive spirometry, increase lung volume and open the airways, preventing atelectasis. Deep breathing also opens the pores of Kohn between alveoli to allow sharing of oxygen between alveoli. Atelectasis can affect anyone who does not deep breathe. A chest tube is for pneumothorax or hemothorax. It is deep breathing, not hyperventilation, that prevents atelectasis.

24. A nurse assesses a client who has mitral valve regurgitation. For which cardiac dysrhythmia should the nurse assess? a. Preventricular contractions b. Atrial fibrillation c. Symptomatic bradycardia d. Sinus tachycardia

ANS: B Atrial fibrillation is a clinical manifestation of mitral valve regurgitation and stenosis. Preventricular contractions and bradycardia are not associated with valvular problems. These are usually identified in clients with electrolyte imbalances, myocardial infarction, and sinus node problems. Sinus tachycardia is a manifestation of aortic regurgitation due to a decrease in cardiac output. DIF: Understanding/Comprehension REF: 692 KEY: Valve disorder| cardiac dysrhythmia MSC: IntegratedProcess:NursingProcess:Assessment NOT: Client Needs Category: Physiological Integrity: Reduction of Risk Potential

8. The home health nurse recommends that a patient with respiratory problems install a carbon monoxide detector in the home. What is the rationale for the nurse's action? a. Carbon monoxide detectors are required by law in the home. b. Carbon monoxide tightly binds to hemoglobin, causing hypoxia. c. Carbon monoxide signals the cerebral cortex to cease ventilations. d. Carbon monoxide combines with oxygen in the body and produces a deadly toxin.

ANS: B Carbon monoxide binds tightly to hemoglobin; therefore, oxygen is not able to bind to hemoglobin and be transported to tissues, causing hypoxia. A carbon monoxide detector is not required by law, does not signal the cerebral cortex to cease ventilations, and does not combine with oxygen but with hemoglobin to produce a toxin.

14. A nurse teaches a client with diabetes mellitus and a body mass index of 42 who is at high risk for coronary artery disease. Which statement related to nutrition would the nurse include in this client's teaching? a. "The best way to lose weight is a high-protein, low-carbohydrate diet." b. "You should balance weight loss with consuming necessary nutrients." c. "A nutritionist will provide you with information about your new diet." d. "If you exercise more frequently, you won't need to change your diet."

ANS: B Clients at risk for cardiovascular diseases should follow the American Heart Association guidelines to combat obesity and improve cardiac health. The nurse would encourage the client to eat vegetables, fruits, unrefined whole-grain products, and fat-free dairy products while losing weight. High-protein food items are often high in fat and calories. Although the nutritionist can assist with client education, the nurse would include nutrition education and assist the client to make healthy decisions. Exercising and eating nutrient-rich foods are both important components in reducing cardiovascular risk.

14. After teaching a client who is being discharged home after mitral valve replacement surgery, the nurse assesses the client's understanding. Which client statement indicates a need for additional teaching? a. "I'll be able to carry heavy loads after 6 months of rest." b. "I will have my teeth cleaned by my dentist in 2 weeks." c. "I must avoid eating foods high in vitamin K, like spinach." d. "I must use an electric razor instead of a straight razor to shave."

ANS: B Clients who have defective or repaired valves are at high risk for endocarditis. The client who has had valve surgery should avoid dental procedures for 6 months because of the risk for endocarditis. When undergoing a mitral valve replacement surgery, the client needs to be placed on anticoagulant therapy to prevent vegetation forming on the new valve. Clients on anticoagulant therapy should be instructed on bleeding precautions, including using an electric razor. If the client is prescribed warfarin, the client should avoid foods high in vitamin K. Clients recovering from open heart valve replacements should not carry anything heavy for 6 months while the chest incision and muscle heal. DIF: Applying/Application REF: 694 KEY: Valve disorder| patient education| hygiene MSC: IntegratedProcess:Teaching/Learning NOT: Client Needs Category: Physiological Integrity: Reduction of Risk Potential

20. A nurse assesses a client who has a history of heart failure. Which question should the nurse ask to assess the extent of the client's heart failure? a. "Do you have trouble breathing or chest pain?" b. "Are you able to walk upstairs without fatigue?" c. "Do you awake with breathlessness during the night?" d. "Do you have new-onset heaviness in your legs?"

ANS: B Clients with a history of heart failure generally have negative findings, such as shortness of breath. The nurse needs to determine whether the client's activity is the same or worse, or whether the client identifies a decrease in activity level. Trouble breathing, chest pain, breathlessness at night, and peripheral edema are symptoms of heart failure, but do not provide data that can determine the extent of the client's heart failure. DIF: Applying/Application REF: 682 KEY: Heart failure| functional ability| respiratory distress/failure MSC: IntegratedProcess:NursingProcess:Assessment NOT: Client Needs Category: Physiological Integrity: Physiological Adaptation

4. A client has a tracheostomy tube in place. When the nurse suctions the client, food particles are noted. What action by the nurse is best? a. Elevate the head of the client's bed. b. Measure and compare cuff pressures. c. Place the client on NPO status. d. Request that the client have a swallow study.

ANS: B Constant pressure from the tracheostomy tube cuff can cause tracheomalacia, leading to dilation of the tracheal passage. This can be manifested by food particles seen in secretions or by noting that larger and larger amounts of pressure are needed to keep the tracheostomy cuff inflated. The nurse should measure the pressures and compare them to previous ones to detect a trend. Elevating the head of the bed, placing the client on NPO status, and requesting a swallow study will not correct this situation.

43. The nurse is educating a student nurse on caring for a patient with a chest tube. Which statement from the student nurse indicates successful learning? a. "I should clamp the chest tube when giving the patient a bed bath." b. "I should report if I see continuous bubbling in the water-seal chamber." c. "I should strip the drains on the chest tube every hour to promote drainage." d. "I should notify the health care provider first, if the chest tube becomes dislodged."

ANS: B Correct care of a chest tube involves knowing normal and abnormal functioning of the tube. A constant or intermittent bubbling in the water-seal chamber indicates a leak in the drainage system, and the health care provider must be notified immediately. Stripping the tube is not routinely performed as it increases pressure. If the tubing disconnects from the drainage unit, instruct the patient to exhale as much as possible and to cough. This maneuver rids the pleural space of as much air as possible. Temporarily reestablish a water seal by immersing the open end of the chest tube into a container of sterile water. The chest tube should not be clamped unless necessary; if so, the length of time clamped would be minimal to reduce the risk of pneumothorax.

12. A nurse assesses a client with mitral valve stenosis. What clinical manifestation should alert the nurse to the possibility that the client's stenosis has progressed? a. Oxygen saturation of 92% b. Dyspnea on exertion c. Muted systolic murmur d. Upper extremity weakness

ANS: B Dyspnea on exertion develops as the mitral valvular orifice narrows and pressure in the lungs increases. The other manifestations do not relate to the progression of mitral valve stenosis. DIF: Applying/Application REF: 688 KEY: Valve disorder| respiratory distress/failure MSC: IntegratedProcess:NursingProcess:Assessment NOT: Client Needs Category: Physiological Integrity: Reduction of Risk Potential

42. The nurse is caring for a patient with a tracheostomy tube. Which nursing intervention is most effective in promoting effective airway clearance? a. Suctioning respiratory secretions several times every hour b. Administering humidified oxygen through a tracheostomy collar c. Instilling normal saline into the tracheostomy to thin secretions before suctioning d. Deflating the tracheostomy cuff before allowing the patient to cough up secretions

ANS: B Humidification from air humidifiers or humidified oxygen tracheostomy collars can help prevent drying of secretions that cause occlusion. Suctioning should be done only as needed; too frequent suctioning can damage the mucosal lining, resulting in thicker secretions. Normal saline should not be instilled into a tracheostomy; research showed no benefit with this technique. The purpose of the tracheostomy cuff is to keep secretions from entering the lungs; the nurse should not deflate the tracheostomy cuff unless instructed to do so by the health care provider.

13. A client has been taking isoniazid for tuberculosis for 3 weeks. What laboratory results need to be reported to the primary health care provider immediately? a. Albumin: 5.1 g/dL (7.4 mcmol/L) b. Alanine aminotransferase (ALT): 180 U/L c. Red blood cell (RBC) count: 5.2/million/μL (5.2 1012/L) d.White blood cell (WBC) count: 12,500/mm3 (12.5 109/L)

ANS: B INH can cause liver damage, especially if the client drinks alcohol. The ALT (one of the liver enzymes) is extremely high and needs to be reported immediately. The albumin and RBCs are normal. The WBCs are slightly high, but that would be an expected finding in a client with an infection.

7. An older adult is brought to the emergency department by a family member, who reports a moderate change in mental status and mild cough. The client is afebrile. The primary health care provider orders a chest x-ray. The family member questions why this is needed since the symptoms seem so vague. What response by the nurse is best? a. "Chest x-rays are always ordered when we suspect pneumonia." b. "Older people often have vague symptoms, so an x-ray is essential." c. "The x-ray can be done and read before laboratory work is reported." d. "We are testing for any possible source of infection in the client."

ANS: B It is essential to obtain an early chest x-ray in older adults suspected of having pneumonia because symptoms are often vague. Waiting until definitive signs and symptoms are present to obtain the x-ray leads to a costly delay in treatment. Stating that chest x-rays are always ordered does not give the family definitive information. The x-ray can be done while laboratory values are still pending, but this also does not provide specific information about the importance of a chest x-ray in this client. The client has symptoms of pneumonia, so the staff is not testing for any possible source of infection but rather is testing for a suspected disorder.

12. A client is wearing a Venturi mask to deliver oxygen and the dinner tray has arrived. What action by the nurse is best? a. Assess the client's oxygen saturation and, if normal, turn off the oxygen. b. Determine if the client can switch to a nasal cannula during the meal. c. Have the client lift the mask off the face when taking bites of food. d. Turn the oxygen off while the client eats the meal and then restart it.

ANS: B Oxygen is a drug that needs to be delivered constantly. The nurse should determine if the provider has approved switching to a nasal cannula during meals. If not, the nurse should consult with the provider about this issue. The oxygen should not be turned off. Lifting the mask to eat will alter the FiO2 delivered.

8. A nurse is caring for a client using oxygen while in the hospital. What assessment finding indicates that goals for a priority diagnosis are being met? a. 100% of meals being eaten by the client b. Intact skin behind the ears c. The client understanding the need for oxygen d. Unchanged weight for the past 3 days

ANS: B Oxygen tubing can cause pressure ulcers, so clients using oxygen have the nursing diagnosis of Risk for Impaired Skin Integrity. Intact skin behind the ears indicates that goals for this diagnosis are being met. Nutrition and weight are not related to using oxygen. Understanding the need for oxygen is important but would not take priority over a physical problem.

5. The charge nurse on a medical unit is preparing to admit several "clients" who have possible pandemic flu during a preparedness drill. What action by the nurse is best? a. Admit the "clients" on Contact Precautions. b. Inquire as to recent travel outside the United States. c. Do not allow pregnant caregivers to care for these "clients." d. Place the "clients" on enhanced Droplet Precautions.

ANS: B Preventing the spread of pandemic flu is equally important as caring for the clients who have it. Preventing the spread of disease is vital. The nurse would ask the "clients" about recent overseas travel to assess the risk of a pandemic flu. Clients with possible pandemic flu need to be in Contact and Airborne Precautions the infectious organism is identified and routes of transmission known. There is no specific danger to pregnant caregivers. Droplet Precautions are not appropriate.

6. A nurse is caring for a patient who was in a motor vehicle accident that resulted in cervical trauma to C4. Which assessment is the priority? a. Pulse b. Respirations c. Temperature d. Blood pressure

ANS: B Respirations and oxygen saturation are the priorities. Cervical trauma at C3 to C5 usually results in paralysis of the phrenic nerve. When the phrenic nerve is damaged, the diaphragm does not descend properly, thus reducing inspiratory lung volumes and causing hypoxemia. While pulse and blood pressure are important, respirations are the priority. Temperature is not a high priority in this situation.

1. A nursing student caring for a client removes the client's oxygen as prescribed. The client is now breathing what percentage of oxygen in the room air? a. 14% b. 21% c. 28% d. 31%

ANS: B Room air is 21% oxygen.

3. A nurse assesses a client admitted to the cardiac unit. Which statement by the client alerts the nurse to the possibility of right-sided heart failure? a. "I sleep with four pillows at night." b. "My shoes fit really tight lately." c. "I wake up coughing every night." d. "I have trouble catching my breath."

ANS: B Signs of systemic congestion occur with right-sided heart failure. Fluid is retained, pressure builds in the venous system, and peripheral edema develops. Left-sided heart failure symptoms include respiratory symptoms. Orthopnea, coughing, and difficulty breathing all could be results of left-sided heart failure. DIF: Understanding/Comprehension REF: 683 KEY: Heart failure| assessment/diagnostic examination MSC: IntegratedProcess:NursingProcess:Assessment NOT: Client Needs Category: Health Promotion and Maintenance

2. A client is scheduled to have a tracheostomy placed in an hour. What action by the nurse is the priority? a. Administer prescribed anxiolytic medication. b. Ensure informed consent is on the chart. c. Reinforce any teaching done previously. d. Start the preoperative antibiotic infusion.

ANS: B Since this is an operative procedure, the client must sign an informed consent, which must be on the chart. Giving anxiolytics and antibiotics and reinforcing teaching may also be required but do not take priority.

45. The nurse is suctioning a patient with a tracheostomy tube. Which action will the nurse take? a. Set suction regulator at 150 to 200 mm Hg. b. Limit the length of suctioning to 10 seconds. c. Apply suction while gently rotating and inserting the catheter. d. Liberally lubricate the end of the suction catheter with a water-soluble solution.

ANS: B Suctioning passes should be limited to 10 seconds to avoid hypoxemia. Suction for a tracheostomy should be set at 100 to 150 mm Hg. Excessive lubrication can clog the catheter or occlude the airway; lubricant is not necessary for oropharyngeal or artificial airway (tracheostomy) suctioning. Suction should never be applied on insertion.

13. A nurse cares for a client recovering from prosthetic valve replacement surgery. The client asks, "Why will I need to take anticoagulants for the rest of my life?" How should the nurse respond? a. "The prosthetic valve places you at greater risk for a heart attack." b. "Blood clots form more easily in artificial replacement valves." c. "The vein taken from your leg reduces circulation in the leg." d. "The surgery left a lot of small clots in your heart and lungs."

ANS: B Synthetic valve prostheses and scar tissue provide surfaces on which platelets can aggregate easily and initiate the formation of blood clots. The other responses are inaccurate. DIF: Applying/Application REF: 696 KEY: Valve disorder| patient education| anticoagulants MSC: IntegratedProcess:Teaching/Learning NOT: Client Needs Category: Physiological Integrity: Reduction of Risk Potential

13. A nurse cares for a client who has an 80% blockage of the right coronary artery (RCA) and is scheduled for bypass surgery. Which intervention would the nurse be prepared to implement while this client waits for surgery? a. Administration of IV furosemide b. Initiation of an external pacemaker c. Assistance with endotracheal intubation d. Placement of central venous access

ANS: B The RCA supplies the right atrium, right ventricle, inferior portion of the left ventricle, and atrioventricular (AV) node. It also supplies the sinoatrial node in 50% of people. If the client totally occludes the RCA, the AV node would not function and the client would go into heart block, so emergency pacing would be available for the client. Furosemide, intubation, and central venous access will not address the primary complication of RCA occlusion, which is AV node (and possibly SA node) malfunction.

10. A client with a new tracheostomy is being seen in the oncology clinic. What finding by the nurse best indicates that goals for the nursing diagnosis Impaired Self-Esteem are being met? a. The client demonstrates good understanding of stoma care. b. The client has joined a book club that meets at the library. c. Family members take turns assisting with stoma care. d. Skin around the stoma is intact without signs of infection.

ANS: B The client joining a book club that meets outside the home and requires him or her to go out in public is the best sign that goals for Impaired Self-Esteem are being met. The other findings are all positive signs but do not relate to this nursing diagnosis.

16. A nurse assesses a client with pericarditis. Which assessment finding should the nurse expect to find? a. Heart rate that speeds up and slows down b. Friction rub at the left lower sternal border c. Presence of a regular gallop rhythm d. Coarse crackles in bilateral lung bases

ANS: B The client with pericarditis may present with a pericardial friction rub at the left lower sternal border. This sound is the result of friction from inflamed pericardial layers when they rub together. The other assessments are not related. DIF: Remembering/Knowledge REF: 699 KEY: Inflammatory response| assessment/diagnostic examination MSC: IntegratedProcess:NursingProcess:Assessment NOT: Client Needs Category: Physiological Integrity: Physiological Adaptation

1. A nurse is teaching staff about the conduction of the heart. In which order will the nurse present the conduction cycle, starting with the first structure? 1. Bundle of His 2. Purkinje network 3. Intraatrial pathways 4. Sinoatrial (SA) node 5. Atrioventricular (AV) node a. 5, 4, 3, 2, 1 b. 4, 3, 5, 1, 2 c. 4, 5, 3, 1, 2 d. 5, 3, 4, 2, 1

ANS: B The conduction system originates with the SA node, the "pacemaker" of the heart. The electrical impulses are transmitted through the atria along intraatrial pathways to the AV node. It assists atrial emptying by delaying the impulse before transmitting it through the Bundle of His and the ventricular Purkinje network.

37. A nurse is caring for a patient with left-sided hemiparesis who has developed bronchitis and has a heart rate of 105 beats/min, blood pressure of 156/90 mm Hg, and respiration rate of 30 breaths/min. Which nursing diagnosis is a priority? a. Risk for skin breakdown b. Impaired gas exchange c. Activity intolerance d. Risk for infection

ANS: B The most important nursing intervention is to maintain airway and circulation for this patient; therefore, Impaired gas exchange is the first nursing priority. Activity intolerance is a concern but is not the priority in this case. Risk for skin breakdown and Risk for infection are also important but do not address an immediate impairment with physiologic integrity.

48. While the nurse is changing the ties on a tracheostomy collar, the patient coughs, dislodging the tracheostomy tube. Which action will the nurse take first? a. Press the emergency response button. b. Insert a spare tracheostomy with the obturator. c. Manually occlude the tracheostomy with sterile gauze. d. Place a face mask delivering 100% oxygen over the nose and mouth.

ANS: B The nurse's first priority is to establish a stable airway by inserting a spare trach into the patient's airway; ideally an obturator should be used. The nurse could activate the emergency response team if the patient is still unstable after the tracheostomy is placed. A patient with a tracheostomy breathes through the tube, not the nose or mouth; a face mask would not be an effective method of getting air into the lungs. Manually occluding pressure over the tracheostomy site is not appropriate and would block the patient's only airway.

18. A nurse teaches a client recovering from a heart transplant who is prescribed cyclosporine (Sandimmune). Which statement should the nurse include in this client's discharge teaching? a. "Use a soft-bristled toothbrush and avoid flossing." b. "Avoid large crowds and people who are sick." c. "Change positions slowly to avoid hypotension." d. "Check your heart rate before taking the medication."

ANS: B These agents cause immune suppression, leaving the client more vulnerable to infection. The medication does not place the client at risk for bleeding, orthostatic hypotension, or a change in heart rate. DIF: Applying/Application REF: 703 KEY: Transplant| immune suppressant MSC: Integrated Process: Nursing Process: Implementation NOT: Client Needs Category: Physiological Integrity: Pharmacological and Parenteral Therapies

14. A client seen in the emergency department reports fever, fatigue, and dry cough but no other upper respiratory symptoms. A chest x-ray reveals mediastinal widening. What action by the nurse is best? a. Collect a sputum sample for culture by deep suctioning. b. Inform the client that oral antibiotics will be needed for 60 days. c. Place the client on Airborne Precautions immediately. d. Tell the client that directly observed therapy is needed.

ANS: B This client has signs and symptoms of early inhalation anthrax. For treatment, after IV antibiotics are finished, oral antibiotics are continued for at least 60 days. Sputum cultures are not needed. Anthrax is not transmissible from person to person, so Standard Precautions are adequate. Directly observed therapy is often used for tuberculosis.

6. A nurse obtains the health history of a client who is newly admitted to the medical unit. Which statement by the client would alert the nurse to the presence of edema? a. "I wake up to go to the bathroom at night." b. "My shoes fit tighter by the end of the day." c. "I seem to be feeling more anxious lately." d. "I drink at least eight glasses of water a day."

ANS: B Weight gain can result from fluid accumulation in the interstitial spaces. This is known as edema. The nurse would note whether the client feels that his or her shoes or rings are tight, and would observe, when present, an indentation around the leg where the socks end. The other answers do not describe edema.

28. Upon auscultation of the patient's chest, the nurse hears a whooshing sound at the fifth intercostal space. What does this finding indicate to the nurse? a. The beginning of the systolic phase b. Regurgitation of the mitral valve c. The opening of the aortic valve d. Presence of orthopnea

ANS: B When regurgitation occurs, there is a backflow of blood into an adjacent chamber. For example, in mitral regurgitation the mitral leaflets do not close completely. When the ventricles contract, blood escapes back into the atria, causing a murmur, or "whooshing" sound. The systolic phase begins with ventricular filling and closing of the aortic valve, which is heard as the first heart sound, S1. Orthopnea is an abnormal condition in which a patient uses multiple pillows when reclining to breathe easier or sits leaning forward with arms elevated.

. A nurse reviews the chart and new prescriptions for a client with diabetic ketoacidosis: Vital Signs and Assessment Laboratory Blood pressure: 90/62 mm Hg Pulse: 120 beats/min Respiratory rate: 28 breaths/min Urine output: 20 mL/hr via catheter Results Serum potassium: 2.6 mEq/L (2.6 mmol/L) Medications Potassium chloride 40 mEq/L (40 mmol/L) IV bolus STAT Increase IV fluid to 100 mL/hr What action would the nurse take? a. Administer the potassium and then consult with the primary health care provider about the fluid prescription. b. Increase the intravenous rate and then consult with the primary health care provider about the potassium prescription. c. Administer the potassium first before increasing the infusion flow rate for the client. d. Increase the intravenous flow rate before administering the potassium to the client.

ANS: B The client is acutely ill and is severely dehydrated and hypokalemic, requiring more IV fluids and potassium. However, potassium would not be infused unless the urine output is at least 30 mL/hr. The nurse would first increase the IV rate and then consult with the primary health care provider about the potassium.

6. A nurse prepares to discharge a client who has heart failure. Based on the national quality measures, which actions should the nurse complete prior to discharging this client? (Select all that apply.) a. Teach the client about energy conservation techniques. b. Ensure the client is prescribed a beta blocker. c. Document a discussion about advanced directives. d. Confirm that a post discharge nurse visit has been scheduled. e. Consult a social worker for additional resources. f. Care transition record transmitted to next level of care within 7 days of discharge.

ANS: B, C, D, F National quality measures aim to decrease heart failure readmission by proper preparation for discharge. These measures include :(1) beta blocker prescribed for left ventricular dysfunction at discharge, (2) postdischarge follow-up appointment scheduled within 7 days of discharge with documentation of location, date, and time. (3) care transition record transmitted to next level of care within 7 days of discharge. (4) documentation of discussion of advance directives/advance care planning with a health care provider, (5) documentation of execution of advance directives within the medical record, and (6) postdischarge evaluation of patient for symptom assessment and treatment adherence within 72 hours of discharge (this can occur by phone, scheduled office visit, or home visit)

9. A nurse assesses a client who is diagnosed with infective endocarditis. Which assessment findings should the nurse expect? (Select all that apply.) a. Weight gain b. Night sweats c. Cardiac murmur d. Abdominal bloating e. Osler's nodes

ANS: B, C, E Clinical manifestations of infective endocarditis include fever with chills, night sweats, malaise and fatigue, anorexia and weight loss, cardiac murmur, and Osler's nodes on palms of the hands and soles of the feet. Abdominal bloating is a manifestation of heart transplantation rejection. DIF: Remembering/Knowledge REF: 697 KEY: Endocarditis MSC: IntegratedProcess:NursingProcess:Assessment NOT: Client Needs Category: Physiological Integrity: Physiological Adaptation

49. A nurse is following the Ventilator Bundle standards to prevent ventilator-associated pneumonia. Which strategies is the nurse using? (Select all that apply.) a. Head of bed elevation to 90 degrees at all times b. Daily oral care with chlorhexidine c. Cuff monitoring for adequate seal d. Clean technique when suctioning e. Daily "sedation vacations" f. Heart failure prophylaxis

ANS: B, C, E The key components of the Institute for Healthcare Improvement (IHI) Ventilator Bundle are: Elevation of the head of the bed (HOB)—elevation is 30 to 45 degrees Daily "sedation vacations" and assessment of readiness to extubate Peptic ulcer disease prophylaxis Deep venous thrombosis prophylaxis Daily oral care with chlorhexidine Monitor cuff pressure frequently to ensure that there is an adequate seal to prevent aspiration of secretions is also included. Sterile technique is used for suctioning when on ventilators. Heart failure prophylaxis is not a component.

A nurse is participating in primary prevention efforts directed against cancer. In which activities is this nurse most likely to engage? (Select all that apply.) a. Demonstrating breast self-examination methods to women b. Instructing people on the use of chemoprevention c. Providing vaccinations against certain cancers d. Screening teenage girls for cervical cancer e. Teaching teens the dangers of tanning booths f. Educating adults about healthy eating habits

ANS: B, C, E, F Primary prevention aims to prevent the occurrence of a disease or disorder, in this case cancer. Secondary prevention includes screening and early diagnosis. Primary prevention activities include teaching people about chemo prevention, providing approved vaccinations to prevent cancer, teaching teens the dangers of tanning beds, and educating adults on eating habits to reduce the risk of getting cancer. Breast examinations and screening for cervical cancer are secondary prevention methods.

The nurse is caring for a client who has diabetes mellitus type 1 and is experiencing hypoglycemia. Which assessment findings will the nurse expect? (Select all that apply.) a. Warm, dry skin b. Nervousness c. Rapid deep respirations d. Dehydration e. Ketoacidosis f. Blurred vision

ANS: B, F The client who has hypoglycemia is often anxious, nervous, and possibly confused. Due to lack of glucose, vision may be blurred or the client may report diplopia (double vision). Clients who have hyperglycemia from diabetes mellitus type 1 have warm skin, Kussmaul respirations that are rapid and deep, dehydration due to elevated blood glucose, and ketoacidosis.

a. Blood urea nitrogen (BUN): 19 mg/dL (6.7 mmol/L) b. International normalized ratio (INR): 6.3 c. Prothrombin time: 35 seconds d. Serum sodium: 130 mEq/L (130 mmol/L) e. White blood cell (WBC) count: 72,000/mm3 (72 109/L)

ANS: B,C Rifampin can cause liver damage, evidenced by the client's high INR and prothrombin time. The BUN and WBC count are normal. The sodium level is low, but that is not related to this client's problem.

3. A nurse assesses a client who is recovering after a coronary catheterization. Which assessment findings in the first few hours after the procedure require immediate action by the nurse? (Select all that apply.) a. Blood pressure of 140/88 mm Hg b. Serum potassium of 2.9 mEq/L (2.9 mmol/L) c. Warmth and redness at the site d. Expanding groin hematoma e. Rhythm changes on the cardiac monitor f. Oxygen saturation 93% on room air

ANS: B,D,E After a cardiac catheterization, the nurse monitors vital signs, entry site, cardiac function, and distal circulation. The potassium is very low which can lead to dysrhythmias. An expanding hematoma signifies bleeding. Rhythm changes on the monitor are a known complication. These findings would require prompt action. The client's blood pressure is slightly elevated but does not need immediate action. Warmth and redness at the site would indicate an infection, but this would not be present in the first few hours. The oxygen saturation is slightly low but not critical and there is no baseline to compare it to.

5. A nurse prepares a client for a pharmacologic stress echocardiogram. What actions would the nurse take when preparing this client for the procedure? (Select all that apply.) a. Assist the primary health care provider to place a central venous access device. b. Prepare for continuous blood pressure and pulse monitoring. c. Administer the client's prescribed beta blocker. d. Give the client nothing by mouth 3 to 6 hours before the procedure. e. Explain to the client that dobutamine will simulate exercise for this examination.

ANS: B,D,E Clients receiving a pharmacologic stress echocardiogram will need peripheral venous access and continuous blood pressure and pulse monitoring. The client must be NPO 3 to 6 hours prior to the procedure. Education about dobutamine, which will be administered during the procedure, would be performed. Beta blockers are often held prior to the procedure as they lower the heart rate and may result in inaccurate results.

A nurse is learning the difference between normal cells and benign tumor cells. What information does this include? a. Benign tumors grow through invasion of other tissue. b. Benign tumors have lost their cellular regulation from contact inhibition. c. Growing in the wrong place or time is typical of benign tumors. d. The loss of characteristics of the parent cells is called anaplasia.

ANS: C Benign tumors are basically normal cells growing in the wrong place or at the wrong time. Benign cells grow through hyperplasia, not invasion. Benign tumor cells retain contact inhibition. Anaplasia is a characteristic of cancer cells.

A nurse has taught a client about dietary changes that can reduce the chances of developing cancer. What statement by the client indicates the nurse needs to provide additional teaching? a. "Foods high in vitamin A and vitamin C are important." b. "I'll have to cut down on the amount of bacon I eat." c. "I'm so glad I don't have to give up my juicy steaks." d. "Vegetables, fruit, and high-fiber grains are important."

ANS: C To decrease the risk of developing cancer, one should cut down on the consumption of red meats and animal fat. The other statements are correct.

12. The nurse suspects the patient has increased afterload. Which piece of equipment should the nurse obtain to determine the presence of this condition? a. Pulse oximeter b. Oxygen cannula c. Blood pressure cuff d. Yankauer suction tip catheter

ANS: C A blood pressure cuff is needed. The diastolic aortic pressure is a good clinical measure of afterload. Afterload is the resistance to left ventricular ejection. In hypertension the afterload increases, making cardiac workload also increase. A pulse oximeter is used to monitor the level of arterial oxygen saturation; it will not help determine increased afterload. While an oxygen cannula may be needed to help decrease the effects of increased afterload, it will not help determine the presence of afterload. A Yankauer suction tip catheter is used to suction the oral cavity.

9. A nurse assesses a client who is recovering after a left-sided cardiac catheterization. Which assessment finding requires immediate intervention? a. Urinary output less than intake b. Bruising at the insertion site c. Slurred speech and confusion d. Discomfort in the left leg

ANS: C A left-sided cardiac catheterization specifically increases the risk for a cerebral vascular accident. A change in neurologic status needs to be acted on immediately. Discomfort and bruising are not unexpected at the site. Urinary output less than intake may or may not be significant.

10. A patient has a myocardial infarction. On which primary blood vessel will the nurse focus care to reduce ischemia? a. Superior vena cava b. Pulmonary artery c. Coronary artery d. Carotid artery

ANS: C A myocardial infarction is the lack of blood flow due to obstruction to the coronary artery, which supplies the heart with blood. The superior vena cava returns blood back to the heart. The pulmonary artery supplies deoxygenated blood to the lungs. The carotid artery supplies blood to the brain.

4. A nurse auscultates heart sounds. When the nurse hears S2, which valves is the nurse hearing close? a. Aortic and mitral b. Mitral and tricuspid c. Aortic and pulmonic d. Mitral and pulmonic

ANS: C As the ventricles empty, the ventricular pressures decrease, allowing closure of the aortic and pulmonic valves, producing the second heart sound, S2. The mitral and tricuspid produce the first heart sound, S1. The aortic and mitral do not close at the same time. The mitral and pulmonic do not close at the same time.

17. After teaching a client who is recovering from a heart transplant to change positions slowly, the client asks, "Why is this important?" How should the nurse respond? a. "Rapid position changes can create shear and friction forces, which can tear out your internal vascular sutures." b. "Your new vascular connections are more sensitive to position changes, leading to increased intravascular pressure and dizziness." c. "Your new heart is not connected to the nervous system and is unable to respond to decreases in blood pressure caused by position changes." d. "While your heart is recovering, blood flow is diverted away from the brain, increasing the risk for stroke when you stand up."

ANS: C Because the new heart is denervated, the baroreceptor and other mechanisms that compensate for blood pressure drops caused by position changes do not function. This allows orthostatic hypotension to persist in the postoperative period. The other options are false statements and do not correctly address the client's question. DIF: Understanding/Comprehension REF: 703 KEY: Transplant| patient education MSC: IntegratedProcess:NursingProcess:Implementation NOT: Client Needs Category: Physiological Integrity: Physiological Adaptation

16. A nurse cares for a client who is recovering from a myocardial infarction. The client states, "I will need to stop eating so much chili to keep that indigestion pain from returning." What is the nurse's best response? a. "Chili is high in fat and calories; it would be a good idea to stop eating it." b. "The primary health care provider has prescribed an antacid every morning." c. "What do you understand about what happened to you?" d. "When did you start experiencing this indigestion?"

ANS: C Clients who experience myocardial infarction often respond with denial, which is a defense mechanism. The nurse would ask the client what he or she thinks happened, or what the illness means to him or her. The other responses do not address the client's misconception about recent pain and the cause of that pain.

1. A nurse working in a geriatric clinic sees clients with "cold" symptoms and rhinitis. The primary health care provider (PHCP) often leaves a prescription for diphenhydramine. What action by the nurse is best? a. Teach the client about possible drowsiness. b. Instruct the client to drink plenty of water. c. Consult with the PHCP about the medication. d. Encourage the client to take the medication with food.

ANS: C First-generation antihistamines are not appropriate for use in the older population. These drugs include chlorpheniramine, diphenhydramine, and hydroxyzine. The nurse would consult with the PHCP and request a different medication. Diphenhydramine does cause drowsiness, but the nurse would request a different medication. Drinking plenty of fluids is appropriate for the condition and is not related to the medication. Antihistamines can be taken without regard to food.

13. The nurse assesses the client using the device pictured below to deliver 50% O2: The nurse finds the mask fits snugly, the skin under the mask and straps is intact, and the flow rate of the oxygen is 3 L/min. What action by the nurse is best? a. Assess the client's oxygen saturation. b. Document these findings in the chart. c. Immediately increase the flow rate. d. Turn the flow rate down to 2 L/min.

ANS: C For the Venturi mask to deliver high flow of oxygen, the flow rate must be set correctly, usually between 4 and 10 L/min. The client's flow rate is too low and the nurse should increase it. After increasing the flow rate, the nurse assesses the oxygen saturation and documents the findings.

7. A nurse assesses female client who is experiencing a myocardial infarction. Which clinical manifestation would the nurse expect? a. Excruciating pain on inspiration b. Left lateral chest wall pain c. Fatigue and shortness of breath d. Numbness and tingling of the arm

ANS: C In women, fatigue, shortness of breath, and indigestion may be the major symptoms of myocardial infarction caused by poor cardiac output. Chest pain is the classic symptom of myocardial infarction and can be present in women. Pain on inspiration may be related to a pleuropulmonary cause. Numbness and tingling of the arm could also be related to the myocardial infarction, but are not known to be specific symptoms for women having and MI.

23. A nurse is caring for a 5-year-old patient whose temperature is 101.2° F. The nurse expects this patient to hyperventilate. Which factor does the nurse remember when planning care for this type of hyperventilation? a. Anxiety over illness b. Decreased drive to breathe c. Increased metabolic demands d. Infection destroying lung tissues

ANS: C Increased body temperature (fever) increases the metabolic rate, thereby increasing carbon dioxide production. The increased carbon dioxide level stimulates an increase in the patient's rate and depth of respiration, causing hyperventilation. Anxiety can cause hyperventilation, but this is not the direct cause from a fever. Sleep causes a decreased respiratory drive; hyperventilation speeds up breathing. The cause of the fever in this question is unknown.

12. A nurse has educated a client on isoniazid. What statement by the client indicates that teaching has been effective? a. "I need to take extra vitamin C while on isoniazid." b. "I should take this medicine with milk or juice." c. "I will take this medication on an empty stomach." d. "My contact lenses will be permanently stained."

ANS: C Isoniazid needs to be taken on an empty stomach, either 1 hour before or 2 hours after meals. Extra vitamin B needs to be taken while on the drug. Staining of contact lenses commonly occurs while taking rifampin.

9. While performing an assessment, the nurse hears crackles in the patient's lung fields. The nurse also learns that the patient is sleeping on three pillows to help with the difficulty breathing during the night. Which condition will the nurse most likely observe written in the patient's medical record? a. Atrial fibrillation b. Myocardial ischemia c. Left-sided heart failure d. Right-sided heart failure

ANS: C Left-sided heart failure results in pulmonary congestion, the signs and symptoms of which include shortness of breath, cough, crackles, and paroxysmal nocturnal dyspnea (difficulty breathing when lying flat). Right-sided heart failure is systemic and results in peripheral edema, weight gain, and distended neck veins. Atrial fibrillation is often described as an irregularly irregular rhythm; rhythm is irregular because of the multiple pacemaker sites. Myocardial ischemia results when the supply of blood to the myocardium from the coronary arteries is insufficient to meet myocardial oxygen demands, producing angina or myocardial infarction.

8. A nurse assesses a client 2 hours after a cardiac angiography via the left femoral artery. The nurse notes that the left pedal pulse is weak. What action would the nurse take next? a. Elevate the leg and apply a sandbag to the entrance site. b. Increase the flow rate of intravenous fluids. c. Assess the color and temperature of the left leg. d. Document the finding as "left pedal pulse of +1/4."

ANS: C Loss of a pulse distal to an angiography entry site is serious, indicating a possible arterial obstruction. The left pulse would be compared with the right, and pulses would be compared with previous assessments, especially before the procedure. Assessing color (pale, cyanosis) and temperature (cool, cold) will identify a decrease in circulation. Once all peripheral and vascular assessment data are acquired, the primary health care provider would be notified. Simply documenting the findings is inappropriate. The leg would be positioned below the level of the heart to increase blood flow to the distal portion of the leg. Increasing intravenous fluids will not address the client's problem.

41. A patient with a pneumothorax has a chest tube inserted and is placed on low constant suction. Which finding requires immediate action by the nurse? a. The patient reports pain at the chest tube insertion site that increases with movement. b. Fifty milliliters of blood gushes into the drainage device after the patient coughs. c. No bubbling is present in the suction control chamber of the drainage device. d. Yellow purulent discharge is seen leaking out from around the dressing site.

ANS: C No bubbling in the suction control chamber indicates an obstruction of the drainage system. An obstruction causes increased pressure, which can cause a tension pneumothorax, which can be life threatening. The nurse needs to determine whether the leak is inside the thorax or in the tubing and act from there. Occasional blood gushes from the lung owing to lung expansion, as during a cough; this is reserve drainage. Drainage over 100 mL/hr after 3 hours of chest tube placement is cause for concern. Yellow purulent drainage indicates an infection that should be reported to the health care provider but is not as immediately life threatening as the lack of bubbling in the suction control chamber.

3. A nurse assesses clients on a medical-surgical unit. Which client would the nurse identify as having the greatest risk for cardiovascular disease? a. An 86-year-old man with a history of asthma. b. A 32-year-old man with colorectal cancer. c. A 65-year-old woman with diabetes mellitus. d. A 53-year-old postmenopausal woman who takes bisphosphonates.

ANS: C Of the options, the client with diabetes has a two- to four-fold increase in risk for death due to cardiovascular disease. Advancing age also increases risk, but not as much. Asthma, colorectal cancer, and bisphosphonate therapy do not increase the risk for cardiovascular disease.

10. A charge nurse is rounding on several older clients on ventilators in the Intensive Care Unit whom the nurse identifies as being at high risk for ventilator-associated pneumonia. To reduce this risk, what activity would the nurse delegate to the assistive personnel (AP)? a. Encourage between-meal snacks. b. Monitor temperature every 4 hours. c. Provide oral care every 4 hours. d. Report any new onset of cough.

ANS: C Oral colonization by gram-negative bacteria is a risk factor for health care-associated pneumonia. Good, frequent oral care can help prevent this from developing and is a task that can be delegated to the AP. Encouraging good nutrition is important, but this will not prevent pneumonia. Monitoring temperature and reporting new cough in clients are important to detect the onset of possible pneumonia but do not prevent it.

4. A nurse assesses an older adult client who has multiple chronic diseases. The client's heart rate is 48 beats/min. What action would the nurse take first? a. Document the finding in the chart. b. Initiate external pacing. c. Assess the client's medications. d. Administer 1 mg of atropine.

ANS: C Pacemaker cells in the conduction system decrease in number as a person ages, potentially resulting in bradycardia. However, the nurse would first check the medication reconciliation for medications that might cause such a drop in heart rate, and then would inform the primary health care provider. Documentation is important, but it is not the first action. The heart rate is not low enough for atropine or an external pacemaker to be needed unless the client is symptomatic, which is not apparent.

26. A nurse is caring for a patient who is taking warfarin (Coumadin) and discovers that the patient is taking garlic to help with hypertension. Which condition will the nurse assess for in this patient? a. Increased cholesterol level b. Distended jugular vein c. Bleeding d. Angina

ANS: C Patients taking warfarin (Coumadin) for anticoagulation prolong the prothrombin time (PT)/international normalized ratio (INR) results if they are taking gingko biloba, garlic, or ginseng with the anticoagulant. The drug interaction can precipitate a life-threatening bleed. Increased cholesterol levels are associated with saturated fat dietary intake. A distended jugular vein and peripheral edema are associated with damage to the right side of the heart. Angina is temporary ischemia of the heart muscle.

2. A nurse is teaching the patient with mitral valve problems about the valves in the heart. Starting on the right side of the heart, describe the sequence of the blood flow through these valves. 1. Mitral 2. Aortic 3. Tricuspid 4. Pulmonic a. 1, 3, 2, 4 b. 4, 3, 2, 1 c. 3, 4, 1, 2 d. 2, 4, 1, 3

ANS: C The blood flows through the valves in the following direction: tricuspid, pulmonic, mitral, and aortic.

38. Which nursing intervention is most effective in preventing hospital-acquired pneumonia in an older-adult patient? a. Discontinue the humidification delivery device to keep excess fluid from lungs. b. Monitor oxygen saturation, and frequently auscultate lung bases. c. Assist the patient to cough, turn, and deep breathe every 2 hours. d. Decrease fluid intake to 300 mL a shift.

ANS: C The goal of the nursing action should be the prevention of pneumonia; the action that best addresses this is to cough, turn, and deep breathe to keep secretions from pooling at the base of the lungs. Humidification thins respiratory secretions, making them easier to expel and should be used. Monitoring oxygen status is important but is not a method of prevention. Hydration assists in preventing hospital-acquired pneumonia. The best way to maintain thin secretions is to provide a fluid intake of 1500 to 2500 mL/day unless contraindicated by cardiac or renal status. Restricting fluids is contraindicated in this situation since there is no data indicating cardiac or renal disease.

17. A nurse prepares a client for cardiac catheterization. The client states, "I am afraid I might die." What is the nurse's best response? a. "This is a routine test and the risk of death is very low." b. "Would you like to speak with a chaplain prior to test?" c. "Tell me more about your concerns about the test." d. "What support systems do you have to assist you?"

ANS: C The nurse would discuss the client's feelings and concerns related to the cardiac catheterization. The nurse would not provide false hope or push the client's concerns off on the chaplain. The nurse would address support systems after addressing the client's current issue.

24. A nurse is preparing a patient for nasotracheal suctioning. In which order will the nurse perform the steps, beginning with the first step? 1. Insert catheter. 2. Apply suction and remove. 3. Have patient deep breathe. 4. Encourage patient to cough. 5. Attach catheter to suction system. 6. Rinse catheter and connecting tubing. a. 1, 2, 3, 4, 5, 6 b. 4, 5, 1, 2, 3, 6 c. 5, 3, 1, 2, 4, 6 d. 3, 1, 2, 5, 4, 6

ANS: C The steps for nasotracheal suctioning are as follows: Verify that catheter is attached to suction; have patient deep breathe; insert catheter; apply intermittent suction for no more than 10 seconds and remove; encourage patient to cough; and rinse catheter and connecting tubing with normal saline.

17. A client is admitted with suspected pneumonia from the emergency department. The client went to the primary health care provider a "few days ago" and shows the nurse the results of what the client calls "an allergy test," as shown below: The reddened area is firm. What action by the nurse is best? a. Assess the client for possible items to which he or she is allergic. b. Call the primary health care provider's office to request records. c. Immediately place the client on Airborne Precautions. d. Prepare to begin administration of intravenous antibiotics.

ANS: C This "allergy test" is actually a positive tuberculosis test. The client would be placed on Airborne Precautions immediately. The other options do not take priority over preventing the spread of the disease.

2. A nurse in a family practice clinic is preparing discharge instructions for a client reporting facial pain that is worse when bending over, tenderness across the cheeks, and postnasal discharge. What instruction will be most helpful? a. "Ice packs may help with the facial pain." b. "Limit fluids to dry out your sinuses." c. "Try warm, moist heat packs on your face." d. "We will schedule a computed tomography scan this week."

ANS: C This client has rhinosinusitis. Comfort measures for this condition include humidification, hot packs, nasal saline irrigations, sleeping with the head elevated, increased fluids, and avoiding cigarette smoke. The client does not need a CT scan.

11. A nurse is teaching a health class about the heart. Which information from the class members indicates teaching by the nurse is successful for the flow of blood through the heart, starting in the right atrium? a. Right ventricle, left ventricle, left atrium b. Left atrium, right ventricle, left ventricle c. Right ventricle, left atrium, left ventricle d. Left atrium, left ventricle, right ventricle

ANS: C Unoxygenated blood flows through the venae cavae into the right atrium, where it is pumped down to the right ventricle; the blood is then pumped out the pulmonary artery and is returned oxygenated via the pulmonary vein to the left atrium, where it flows to the left ventricle and is pumped out to the rest of the body via the aorta.

14. A patient's heart rate increased from 94 to 164 beats/min. What will the nurse expect? a. Increase in diastolic filling time b. Decrease in hemoglobin level c. Decrease in cardiac output d. Increase in stroke volume

ANS: C With a sustained heart rate greater than 160 beats/min, diastolic filling time decreases, decreasing stroke volume and cardiac output. The hemoglobin level would not be affected.

51. A nurse is caring for a patient with sleep apnea. Which types of ventilator support should the nurse be prepared to administer for this patient? (Select all that apply.) a. Assist-control (AC) b. Pressure support ventilation (PSV) c. Bilevel positive airway pressure (BiPAP) d. Continuous positive airway pressure (CPAP) e. Synchronized intermittent mandatory ventilation (SIMV)

ANS: C, D Ventilatory support is achieved using a variety of modes, including continuous positive airway pressure (CPAP) and bilevel positive airway pressure (BiPAP). The purpose of CPAP and BiPAP is to maintain a positive airway pressure and improve alveolar ventilation. This prevents or treats atelectasis by inflating the alveoli, reducing pulmonary edema by forcing fluid out of the lungs back into circulation, and improving oxygenation in those with sleep apnea. AC, PSV, and SIMV are invasive mechanical ventilation and are not routinely used on patients with sleep apnea. AC delivers a set tidal volume (VT) with each breath, regardless of whether the breath was triggered by the patient or the ventilator. Synchronized intermittent mandatory ventilation like AC delivers a minimum number of fully assisted breaths per minute that are synchronized with the patient's respiratory effort. Any breaths taken between volume-cycled breaths are not assisted; the volume of these breaths is determined by the patient's strength, effort, and lung mechanics. PSV mode is often combined with SIMV mode: inspiratory pressure is added to spontaneous breaths to overcome the resistance of the endotracheal tube or to help increase the volume of the patient's spontaneous breaths.

5. The emergency department nurse is participating in a bioterrorism drill in which several "clients" are suspected to have inhalation anthrax. Which "clients" would the nurse see as the priorities? (Select all that apply.) a. Widened mediastinum on chest x-ray b. Dry cough c. Stridor d. Oxygen saturation of 91% e. Diaphoresis f. Oral temperature of 99.9° F (37.7° C)

ANS: C,D,E Clients with fulminant anthrax may exhibit stridor, hypoxia, and diaphoresis. Although an oxygen saturation of 91% is not critical, it is anormally low. These clients would be seen as the priority. A widened mediastinum and dry cough are usually seen in the prodromal phase when the temperature elevation is not as severe.

A nurse is assessing a client with glioblastoma. What assessment is most important? a. Abdominal palpation b. Abdominal percussion c. Lung auscultation d. Neurologic examination

ANS: D A glioblastoma arises in the brain. The most important assessment for this client is the neurologic examination.

A nurse asks the staff development nurse what "apoptosis" means. What response best? a. Growth by cells enlarging b. Having the normal number of chromosomes c. Inhibition of cell growth d. Programmed cell death

ANS: D Apoptosis is programmed cell death. With this characteristic, organs and tissues function with cells that are at their peak of performance. Growth by cells enlarging is hyperplasia. Having the normal number of chromosomes is euploidy. Inhibition of cell growth is contact inhibition.

A client is in the oncology clinic for a first visit since being diagnosed with cancer. The nurse reads in the client's chart that the cancer classification is TISN0M0. What does the nurse conclude about this client's cancer? a. The primary site of the cancer cannot be determined. b. Regional lymph nodes could not be assessed. c. There are multiple lymph nodes involved already. d. There are no distant metastases noted in the report.

ANS: D TIS stands for carcinoma in situ; N0 stands for no regional lymph node metastasis; and M0 stands for no distant metastasis.

8. A nurse assesses a client after administering isosorbide mononitrate (Imdur). The client reports a headache. Which action should the nurse take? a. Initiate oxygen therapy. b. Hold the next dose of Imdur. c. Instruct the client to drink water. d. Administer PRN acetaminophen.

ANS: D The vasodilating effects of isosorbide mononitrate frequently cause clients to have headaches during the initial period of therapy. Clients should be told about this side effect and encouraged to take the medication with food. Some clients obtain relief with mild analgesics, such as acetaminophen. The client's headache is not related to hypoxia or dehydration; therefore, these interventions would not help. The client needs to take the medication as prescribed to prevent angina; the medication should not be held. DIF: Applying/Application REF: 686 KEY: Heart failure| nitroglycerin/nitrates| medication| pharmacologic pain management MSC: Integrated Process: Nursing Process: Implementation NOT: Client Needs Category: Physiological Integrity: Pharmacological and Parenteral Therapies

Which statement about carcinogenesis is accurate? a. An initiated cell will always become clinical cancer. b. Cancer becomes a health problem once it is 1 cm in size. c. Normal hormones and proteins do not promote cancer growth. d. Tumor cells need to develop their own blood supply.

ANS: D Tumors need to develop their own blood supply through a process called angiogenesis. An initiated cell needs a promoter to continue its malignant path. Normal hormones and proteins in the body can act as promoters. A 1-cm tumor is a detectable size, but other events have to occur for it to become a health problem.

18. An emergency department nurse triages clients who present with chest discomfort. Which client would the nurse plan to assess first? a. Client who describes pain as a dull ache. b. Client who reports moderate pain that is worse on inspiration. c. Client who reports cramping substernal pain. d. Client who describes intense squeezing pressure across the chest.

ANS: D All clients who have chest pain would be assessed more thoroughly. To determine which client would be seen first, the nurse must understand common differences in pain descriptions. Intense stabbing and viselike (squeezing) substernal pain or pressure that spreads through the client 's chest, arms, jaw, back, or neck are indicatives of a myocardial infarction. The nurse would plan to see this client first to prevent cardiac cell death. A dull ache, pain that gets worse with inspiration, and cramping pain are not usually associated with myocardial infarction.

10. A nurse assesses a client who is scheduled for a cardiac catheterization. Which assessment would the nurse complete as the priority prior to this procedure? a. Client's level of anxiety b. Ability to turn self in bed c. Cardiac rhythm and heart rate d. Allergies to iodine-based agents

ANS: D Before the procedure, the nurse would ascertain whether the client has an allergy to iodine-containing preparations, such as seafood or local anesthetics. The contrast medium used during the procedure is iodine based. This allergy can cause a life-threatening reaction, so it is a high priority. It is important for the nurse to assess anxiety, mobility, and baseline cardiac status, but allergies take priority for client safety.

2. A nurse assesses a client after administering a prescribed beta blocker. Which assessment would the nurse expect to find? a. Blood pressure increased from 98/42 to 132/60 mm Hg. b. Respiratory rate decreased from 25 to 14 breaths/min. c. Oxygen saturation increased from 88% to 96%. d. Pulse decreased from 100 to 80 beats/min.

ANS: D Beta blockers block the stimulation of beta1-adrenergic receptors. They block the sympathetic (fight-or-flight) response and decrease the heart rate (HR). The beta blocker will decrease HR and blood pressure, increasing ventricular filling time. It usually does not have effects on beta2-adrenergic receptor sites. Cardiac output may drop because of decreased HR, but slowing the rate may allow for better filling and better cardiac output.

10. A nurse teaches a client who has a history of heart failure. Which statement should the nurse include in this client's discharge teaching? a. "Avoid drinking more than 3 quarts of liquids each day." b. "Eat six small meals daily instead of three larger meals." c. "When you feel short of breath, take an additional diuretic." d. "Weigh yourself daily while wearing the same amount of clothing."

ANS: D Clients with heart failure are instructed to weigh themselves daily to detect worsening heart failure early, and thus avoid complications. Other signs of worsening heart failure include increasing dyspnea, exercise intolerance, cold symptoms, and nocturia. Fluid overload increases symptoms of heart failure. The client should be taught to eat a heart-healthy diet, balance intake and output to prevent dehydration and overload, and take medications as prescribed. The most important discharge teaching is daily weights as this provides the best data related to fluid retention. DIF: Applying/Application REF: 687 KEY: Heart failure| patient education MSC: Integrated Process: Teaching/Learning NOT: Client Needs Category: Health Promotion and Maintenance

47. The nurse is using a closed suction device. Which patient will be most appropriate for this suctioning method? a. A 5-year-old with excessive drooling from epiglottitis b. A 5-year-old with an asthma attack following severe allergies c. A 24-year-old with a right pneumothorax following a motor vehicle accident d. A 24-year-old with acute respiratory distress syndrome requiring mechanical ventilation

ANS: D Closed suctioning is most often used on patients who require invasive mechanical ventilation to support their respiratory efforts because it permits continuous delivery of oxygen while suction is performed and reduces the risk of oxygen desaturation. In this case, the acute respiratory distress syndrome requires mechanical ventilation. In the presence of epiglottitis, croup, laryngospasm, or irritable airway, the entrance of a suction catheter via the nasal route causes intractable coughing, hypoxemia, and severe bronchospasm, necessitating emergency intubation or tracheostomy. The 5-year-old child with asthma would benefit from an inhaler. A chest tube is needed for the pneumothorax.

22. The nurse is caring for a patient with respiratory problems. Which assessment finding indicates a late sign of hypoxia? a. Elevated blood pressure b. Increased pulse rate c. Restlessness d. Cyanosis

ANS: D Cyanosis, blue discoloration of the skin and mucous membranes caused by the presence of desaturated hemoglobin in capillaries, is a late sign of hypoxia. Elevated blood pressure, increased pulse rate, and restlessness are early signs of hypoxia.

5. The nurse is teaching about the process of exchanging gases through the alveolar capillary membrane. Which term will the nurse use to describe this process? a. Ventilation b. Surfactant c. Perfusion d. Diffusion

ANS: D Diffusion is the process of gases exchanging across the alveoli and capillaries of body tissues.Ventilation is the process of moving gases into and out of the lungs. Surfactant is a chemical produced in the lungs to maintain the surface tension of the alveoli and keep them from collapsing. Perfusion is the ability of the cardiovascular system to carry oxygenated blood to tissues and return deoxygenated blood to the heart.

16. A client is being discharged on long-term therapy for tuberculosis (TB). What referral by the nurse is most appropriate? a. Community social worker for Meals on Wheels b. Occupational therapy for job retraining c. Physical therapy for homebound therapy services d. Visiting nurses for directly observed therapy

ANS: D Directly observed therapy is often utilized for managing clients with TB in the community. Meals on Wheels, job retraining, and home therapy may or may not be appropriate.

9. A nurse teaches a client who is prescribed digoxin (Lanoxin) therapy. Which statement should the nurse include in this client's teaching? a. "Avoid taking aspirin or aspirin-containing products." b. "Increase your intake of foods that are high in potassium." c. "Hold this medication if your pulse rate is below 80 beats/min." d. "Do not take this medication within 1 hour of taking an antacid."

ANS: D Gastrointestinal absorption of digoxin is erratic. Many medications, especially antacids, interfere with its absorption. Clients are taught to hold their digoxin for bradycardia; a heart rate of 80 beats/min is too high for this cutoff. Potassium and aspirin have no impact on digoxin absorption, nor do these statements decrease complications of digoxin therapy. DIF: Applying/Application REF: 686 KEY: Heart failure| digoxin| medication| patient education MSC: Integrated Process: Nursing Process: Implementation NOT: Client Needs Category: Physiological Integrity: Pharmacological and Parenteral Therapies

39. The nurse is assessing a patient with emphysema. Which assessment finding requires further follow-up with the health care provider? a. Increased anterior-posterior diameter of the chest b. Accessory muscle used for breathing c. Clubbing of the fingers d. Hemoptysis

ANS: D Hemoptysis is an abnormal occurrence of emphysema, and further diagnostic studies are needed to determine the cause of blood in the sputum. Clubbing of the fingers, barrel chest (increased anterior-posterior chest diameter), and accessory muscle use are all normal findings in a patient with emphysema.

9. A client has been admitted for suspected inhalation anthrax infection. What question by the nurse is most important? a. "Are any family members also ill?" b. "Have you traveled recently?" c. "How long have you been ill?" d. "What is your occupation?"

ANS: D Inhalation anthrax is rare and is an occupational hazard among people who work with animal wool, bone meal, hides, and skin, such as taxidermists and veterinarians. Inhalation anthrax seen in someone without an occupational risk is considered a bioterrorism event and must be reported to authorities immediately. The other questions are appropriate for anyone with an infection.

23. A nurse is caring for a client with acute pericarditis who reports substernal precordial pain that radiates to the left side of the neck. Which nonpharmacologic comfort measure should the nurse implement? a. Apply an ice pack to the client's chest. b. Provide a neck rub, especially on the left side. c. Allow the client to lie in bed with the lights down. d. Sit the client up with a pillow to lean forward on.

ANS: D Pain from acute pericarditis may worsen when the client lays supine. The nurse should position the client in a comfortable position, which usually is upright and leaning slightly forward. Pain is decreased by using gravity to take pressure off the heart muscle. An ice pack and neck rub will not relieve this pain. DIF: Applying/Application REF: 699 KEY: Nonpharmacologic pain management MSC: Integrated Process: Nursing Process: Implementation NOT: Client Needs Category: Physiological Integrity: Basic Care and Comfort

17. A patient has inadequate stroke volume related to decreased preload. Which treatment does the nurse prepare to administer? a. Diuretics b. Vasodilators c. Chest physiotherapy d. Intravenous (IV) fluids

ANS: D Preload is affected by the circulating volume; if the patient has decreased fluid volume, it will need to be replaced with fluid or blood therapy. Preload is the amount of blood in the left ventricle at the end of diastole, often referred to as end-diastolic volume. Giving diuretics and vasodilators will make the situation worse. Diuretics causes fluid loss; the patient is already low on fluids or the preload would not be decreased. Vasodilators reduced blood return to the heart, making the situation worse; the patient does not have enough blood and fluid to the heart or the preload would not be decreased. Chest physiotherapy is a group of therapies for mobilizing pulmonary secretions. Chest physiotherapy will not help this cardiovascular problem.

4. A client is in the family practice clinic reporting a severe "cold" that started 4 days ago. On examination, the nurse notes that the client also has a severe headache and muscle aches. What action by the nurse is best? a. Educate the client on oseltamivir. b. Facilitate admission to the hospital. c. Instruct the client to have a flu vaccine. d. Teach the client to sneeze in the upper sleeve.

ANS: D Sneezing and coughing into one's sleeve helps prevent the spread of upper respiratory infections. The client does have symptoms of the flu (influenza), but it is too late to start antiviral medications; to be effective, they must be started within 24 to 48 hours of symptom onset. The client does not need hospital admission. The client would be instructed to have a flu vaccination, but now that he or she has the flu, vaccination will have to wait until next year.

18. A nurse is preparing to suction a patient. The pulse is 65 and pulse oximetry is 94%. Which finding will cause the nurse to stop suctioning? a. Pulse 75 b. Pulse 80 c. Oxygen saturation 91% d. Oxygen saturation 88%

ANS: D Stop when oxygen saturation is 88%. Monitor patient's vital signs and oxygen saturation during procedure; note whether there is a change of 20 beats/min (either increase or decrease) or if pulse oximetry falls below 90% or 5% from baseline. If this occurs, stop suctioning. A pulse rate of 75 is only 10 beats different from the start of the procedure. A pulse rate of 80 is 15 beats different from the start of suctioning. Oxygen saturation of 91% is not 5% from baseline or below 90%.

15. A nurse cares for a client who has advanced cardiac disease and states, "I am having trouble breathing while I'm sleeping at night." What is the nurse's best response? a. "I will consult your primary health care provider to prescribe a sleep study." b. "You become hypoxic while sleeping; oxygen therapy via nasal cannula will help." c. "A continuous positive airway pressure, or CPAP, breathing mask will help you breathe at night." d. "Use pillows to elevate your head and chest while you are sleeping."

ANS: D The client is experiencing orthopnea (shortness of breath while lying flat). The nurse would teach the client to elevate the head and chest with pillows or sleep in a recliner. A sleep study is not necessary to diagnose this client. Oxygen and CPAP will not help a client with orthopnea.

19. A nurse cares for a client with end-stage heart failure who is awaiting a transplant. The client appears depressed and states, "I know a transplant is my last chance, but I don't want to become a vegetable." How should the nurse respond? a. "Would you like to speak with a priest or chaplain?" b. "I will arrange for a psychiatrist to speak with you." c. "Do you want to come off the transplant list?" d. "Would you like information about advance directives?"

ANS: D The client is verbalizing a real concern or fear about negative outcomes of the surgery. This anxiety itself can have a negative effect on the outcome of the surgery because of sympathetic stimulation. The best action is to allow the client to verbalize the concern and work toward a positive outcome without making the client feel as though he or she is crazy. The client needs to feel that he or she has some control over the future. The nurse personally provides care to address the client's concerns instead of pushing the client's issues off on a chaplain or psychiatrist. The nurse should not jump to conclusions and suggest taking the client off the transplant list, which is the best treatment option. DIF: Applying/Application REF: 691 KEY: Transplant| psychosocial response| anxiety MSC: Integrated Process: Nursing Process: Implementation NOT: Client Needs Category: Psychosocial Integrity

9. A nurse is assessing a client who has a tracheostomy. The nurse notes that the tracheostomy tube is pulsing with the heartbeat as the client's pulse is being taken. No other abnormal findings are noted. What action by the nurse is most appropriate? a. Call the operating room to inform them of a pending emergency case. b. No action is needed at this time; this is a normal finding in some clients. c. Remove the tracheostomy tube; ventilate the client with a bag-valve-mask. d. Stay with the client and have someone else call the provider immediately.

ANS: D This client may have a trachea-innominate artery fistula, which can be a life-threatening emergency if the artery is breached and the client begins to hemorrhage. Since no bleeding is yet present, the nurse stays with the client and asks someone else to notify the provider. If the client begins hemorrhaging, the nurse removes the tracheostomy and applies pressure at the bleeding site. The client will need to be prepared for surgery.

The nurse is assessing a client for risk of developing metabolic syndrome. Which risk factor is associated with this health condition? a. Hypotension b. Hyperthyroidism c. Abdominal obesity d. Hypoglycemia

Abdominal obesity

A nurse assesses a client who has diabetes mellitus and notes that the client is awake and alert, but shaky, diaphoretic, and weak. Five minutes after administering a half-cup (120 mL) of orange juice, the client's signs and symptoms have not changed. What action would the nurse take next? a. Administer another half-cup (120 mL) of orange juice. b. Administer a half-ampule of dextrose 50% intravenously. c. Administer 10 units of regular insulin subcutaneously. d. Administer 1 mg of glucagon intramuscularly.

Administer another half-cup (120 mL) of orange juice.

The nurse is preparing to teach a community group about warning signs of lung cancer. What information does the nurse include? (SATA) a. Over 10 pack year history of smoking b. Persistent coughing c. Rusty or blood-tinged sputum d. Dyspnea e. Hoarseness f. Fatigue

B, C, D, E Some common signs of lung cancer include persistent cough, rusty or blood-tinged sputum, dyspnea, and hoarseness. Fatigue is common to many conditions. Smoking history is a risk factor for lung cancer.

A nurse is teaching a client how to perform pursed-lip breathing. Which instruction would the nurse include in this teaching? (SATA) a. Open your mouth and breathe deeply b. Use your abdominal muscles to squeeze air out of your lungs c. Breath out slowly without puffing your cheeks d. Focus on inhaling and holding your breath as long as you can e. Exhale at least twice the amount of time it took to breathe in f. Lie on your back with your knees bent

B, C, E A nurse would teach a client to close his or her mouth and breathe in through his or her nose, purse his or her lips and breathe out slowly without puffing his or her cheeks, and use his or her abdominal muscles to squeeze out every bit of air. The nurse would also remind the client to use pursed-lip breathing during any physical activity, to focus on exhaling, and to never hold his or her breath. Lying on the back with bent knees is the preferred position for diaphragmatic breathing.

A nurse assesses a client with COPD. Which questions would the nurse ask to determine the client's activity tolerance? (SATA) a. What color is your sputum? b. Do you have any difficulty sleeping? c. How long does it take to perform your morning routine? d. Do you walk upstairs everyday? e. Have you lost any weight lately? f. How does your activity compare to this time last year?

B, C, E, F Difficulty sleeping could indicate worsening breathlessness, as could taking longer to perform ADLs. Weight loss could mean increased dyspnea as the client becomes too fatigued to eat. The color of the client's sputum would not assist in determining activity tolerance. Asking whether the client walks upstairs every day is not as pertinent as determining if the client becomes short of breath on walking upstairs, or if the client goes upstairs less often than previously. The nurse would ask the client to compare his or her current level of activity with that of a month or even a year ago.

A nurse assesses a client who has a mediastinal chest tube. Which symptoms require the nurse's IMMEDIATE intervention? (SATA) a. Production of pink sputum b. Tracheal deviation c. Pain at insertion site d. Sudden onset of shortness of breath e. Drainage greater than 70 mL/hr f. Disconnection at Y site

B, D, E, F Immediate intervention is warranted if the client has tracheal deviation because this could indicate a tension pneumothorax. Sudden shortness of breath could indicate dislodgement of the tube, occlusion of the tube, or pneumothorax. Drainage greater than 70 mL/hr could indicate hemorrhage. Disconnection at the Y site could result in air entering the tubing. Production of pink sputum and pain at the insertion site are not signs/symptoms that would require immediate attention.

A pulmonary nurse cares for clients who have COPD. Which client would the nurse assess FIRST? a. A 46 year old with a 30 pack history of smoking b. A 52 year old in a tripod position using accessory muscles to breath c. A 68 year old who has dependent edema and clubbed fingers d. A 74 year old with a chronic cough and thick, tenacious secretions

B. A 52 year old in a tripod position using accessory muscles to breath The client who is in a tripod position and using accessory muscles is working to breathe. This client must be assessed first to establish how effectively the client is breathing and provide interventions to minimize respiratory distress. The other clients are not in acute distress.

A client has received a dose of ondansetron (Zofran) for nausea. What action by the nurse is most important? A. Assess the client for a headache. B. Assist the client in getting out of bed. C. Instruct the client to reduce salt intake. D. Weigh the client daily before the client eats.

B. Assist the client in getting out of bed. Rationale: Ondansetron side effects include postural hypotension, vertigo, and bradycardia, all of which increase the clients risk for injury. The nurse should assist the client when getting out of bed. Headache and fluid retention are not side effects of this drug.

A nurse administers medications to a client who has asthma. Which medication classification is paired correctly with its physiologic action? a. Bronchodilator - stabilizes the membranes of mast cells and prevents the release of inflammatory mediators b. Cholinergic antagonist - causes bronchodilation by inhibiting the parasympathetic nervous system c. Corticosteroid - relaxes bronchiolar smooth muscles by binding to and activating pulmonary beta2 receptors d. Cromone - disrupts the production of pathways of inflammatory mediators

B. Cholinergic antagonist - causes bronchodilation by inhibiting the parasympathetic nervous system Cholinergic antagonist drugs cause bronchodilation by inhibiting the parasympathetic nervous system. This allows the SNS to dominate and release norepinephrine that activates beta2 receptors. Bronchodilators relax bronchiolar smooth muscles by binding to and activating pulmonary beta2 receptors. Corticosteroids disrupt the production of pathways of inflammatory mediators. Cromones stabilize the membranes of mast cells and prevent the release of inflammatory mediators.

While assessing a client who is 12 hours postoperative after a thoracotomy for lung cancer, a nurse notices that the chest tube is dislodged. What action by the nurse is BEST? a. Assess for drainage from the sight b. Cover the insertion site with sterile gauze c. Contact the primary health care provider d. Reinsert the tube using sterile technique

B. Cover the insertion site with sterile gauze Immediately covering the insertion site helps prevent air from entering the pleural space and causing a pneumothorax. The area will not reseal quickly enough to prevent air from entering the chest. The nurse would not leave the patient to obtain a suture kit. An occlusive dressing may cause a tension pneumothorax. The nurse does not need to assess the site at this moment. The primary health care provider would be called to reinsert the chest tube or prescribe other treatment options.

A client tells the oncology nurse about an upcoming vacation to the beach to celebrate completing radiation treatments for cancer. What response by the nurse is most appropriate? A. Avoid getting salt water on the radiation site. B. Do not expose the radiation area to direct sunlight. C. Have a wonderful time and enjoy your vacation! D. Remember you should not drink alcohol for a year.

B. Do not expose the radiation area to direct sunlight. Rationale: The skin overlying the radiation site is extremely sensitive to sunlight after radiation therapy has been completed. The nurse should inform the client to avoid sun exposure to this area. This advice continues for 1 year after treatment has been completed. The other statements are not appropriate.

A client is having a catheter placed in the femoral artery to deliver yttrium-90 beads into a liver tumor. What action by the nurse is most important? A. Assessing the clients abdomen beforehand B. Ensuring that informed consent is on the chart C. Marking the clients bilateral pedal pulses D. Reviewing client teaching done previously

B. Ensuring that informed consent is on the chart Rationale: This is an invasive procedure requiring informed consent. The nurse should ensure that consent is on the chart. The other actions are also appropriate but not the priority.

A client with cancer has anorexia and mucositis, and is losing weight. The clients family members continually bring favorite foods to the client and are distressed when the client wont eat them. What action by the nurse is best? A. Explain the pathophysiologic reasons behind the client not eating. B. Help the family show other ways to demonstrate love and caring. C. Suggest foods and liquids the client might be willing to try to eat. D. Tell the family the client isn't able to eat now no matter what they bring.

B. Help the family show other ways to demonstrate love and caring. Rationale: Families often become distressed when their loved ones wont eat. Providing food is a universal sign of caring, and to some people the refusal to eat signifies worsening of the condition. The best option for the nurse is to help the family find other ways to demonstrate caring and love, because with treatment-related anorexia and mucositis, the client is not likely to eat anything right now. Explaining the rationale for the problem is a good idea but does not suggest to the family anything that they can do for the client. Simply telling the family the client is not able to eat does not give them useful information and is dismissive of their concerns.

A clinic nurse is reviewing care measures with a client who has asthma, Step 3. What statement by the client indicates the need to REVIEW the information? a. I still will use my rapid-acting inhaler for an asthma attack b. I will always use the spacer with my dry powder inhaler c. If I am stable for 3 months, I might be able to reduce my drugs d. My inhaled corticosteroid must be taken regularly to work well

B. I will always use the spacer with my dry powder inhaler Dry powder inhalers are not used with a spacer. The other statements are accurate.

The student nurse caring for clients who have cancer understands that the general consequences of cancer include which client problems? (Select all that apply.) A. Clotting abnormalities from thrombocythemia B. Increased risk of infection from white blood cell deficits C. Nutritional deficits such as early satiety and cachexia D. Potential for reduced gas exchange E. Various motor and sensory deficits

B. Increased risk of infection from white blood cell deficits C. Nutritional deficits such as early satiety and cachexia D. Potential for reduced gas exchange E. Various motor and sensory deficits Rationale: The general consequences of cancer include reduced immunity and blood-producing functions, altered GI structure and function, decreased respiratory function, and motor and sensory deficits. Clotting problems often occur due to thrombocytopenia (not enough platelets), not thrombocythemia (too many platelets).

A client has a platelet count of 9800/mm3. What action by the nurse is most appropriate? A. Assess the client for calf pain, warmth, and redness. B. Instruct the client to call for help to get out of bed. C. Obtain cultures as per the facilitys standing policy. D. Place the client on protective isolation precautions.

B. Instruct the client to call for help to get out of bed. Rationale: A client with a platelet count this low is at high risk for serious bleeding episodes. To prevent injury, the client should be instructed to call for help prior to getting out of bed. Calf pain, warmth, and redness might indicate a deep vein thrombosis, not associated with low platelets. Cultures and isolation relate to low white cell counts.

A client in the oncology clinic reports her family is frustrated at her ongoing fatigue 4 months after radiation therapy for breast cancer. What response by the nurse is most appropriate? A. Are you getting adequate rest and sleep each day? B. It is normal to be fatigued even for years afterward. C. This is not normal and Ill let the provider know. D. Try adding more vitamins B and C to your diet.

B. It is normal to be fatigued even for years afterward. Rationale: Regardless of the cause, radiation-induced fatigue can be debilitating and may last for months or years after treatment has ended. Rest and adequate nutrition can affect fatigue, but it is most important that the client understands this is normal.

After receiving the hand-off report, which client should the oncology nurse see first? A. Client who is afebrile with a heart rate of 108 beats/min B. Older client on chemotherapy with mental status changes C. Client who is neutropenic and in protective isolation D. Client scheduled for radiation therapy today

B. Older client on chemotherapy with mental status changes Rationale: Older clients often do not exhibit classic signs of infection, and often mental status changes are the first observation. Clients on chemotherapy who become neutropenic also often do not exhibit classic signs of infection. The nurse should assess the older client first. The other clients can be seen afterward.

After teaching a client how to perform diaphragmatic breathing, the nurse assesses the client's understanding. Which action demonstrates that the client CORRECTLY understands the teaching? a. The client lies on his or her side with knees bent b. The client places his or her hands on the abdomen c. The client lies in a prone position d. The client places his or her hands above the head

B. The client places his or her hands on the abdomen To perform diaphragmatic breathing correctly, the client would place his or her hands on the abdomen to create resistance. This type of breathing cannot be performed effectively while lying on the side or with hands over the head. This type of breathing would not be as effective lying prone.

A nurse assesses a client with asthma and notes bilateral wheezing, decreased pulse oxygen saturation, and suprasternal retraction on inhalation. What actions by the nurse are BEST? (SATA) a. Administer prescribed salmeterol inhaler b. Assess the client for a tracheal deviation c. Administer oxygen and place client on an oximeter d. Perform peak expiratory flow readings e. Administer prescribed albuterol inhaler f. Assess the client's lung sounds after administering the inhaler

C, E, F Suprasternal retraction caused by inhalation usually indicates that the client is using accessory muscles and is having difficulty moving air into the respiratory passages because of airway narrowing. Wheezing indicates a narrowed airway; a decreased pulse oxygen saturation also supports this finding. The asthma is becoming unstable, and intervention is needed. Administration of a rescue inhaler is indicated, probably along with administration of oxygen. The nurse would reassess lung sounds after the rescue inhaler. The nurse would not do a peak flow reading at this time, nor would a code be called. The nurse could assess for tracheal deviation after administering oxygen and albuterol.

Four clients are receiving tyrosine kinase inhibitors (TKIs). Which of these four clients should the nurse assess first? A. Client with dry, itchy, peeling skin B. Client with a serum calcium of 9.2 mg/dL C. Client with a serum potassium of 2.8 mEq/L D. Client with a weight gain of 0.5 pound (1.1 kg) in 1 day

C. Client with a serum potassium of 2.8 mEq/L Rationale: TKIs can cause electrolyte imbalances. This potassium level is very low, so the nurse should assess this client first. Dry, itchy, peeling skin can be a problem in clients receiving biologic response modifiers, and the nurse should assess that client next because of the potential for discomfort and infection. This calcium level is normal. TKIs can also cause weight gain, but the client with the low potassium level is more critical.

A nurse cares for a client with chronic obstructive pulmonary disease (COPD) who appears thin and disheveled. Which question would the nurse ask FIRST? a. Do you have a strong support system? b. What do you understand about your disease? c. Do you experience shortness of breath with basic activities? d. What medications are you prescribed to take each day?

C. Do you experience shortness of breath with basic activities? Clients with severe COPD may not be able to perform daily activities, including bathing and eating, because of excessive shortness of breath. The nurse would ask the client if shortness of breath is interfering with basic activities. Although the nurse would need to know about the client's support systems, current knowledge, and medications, these questions do not address the client's appearance.

A nurse is teaching a client who has cystic fibrosis (CF). Which statement would the nurse include in this client's teaching? a. Take an antibiotic each day b. You should get genetic screening c. Eat a well-balanced, nutritious diet d. Plan to exercise for 30 minutes every day

C. Eat a well-balanced, nutritious diet Clients with CF often are malnourished due to vitamin deficiency and pancreatic malfunction. Maintaining nutrition is essential. Daily antibiotics and daily exercise are not essential actions. Genetic screening might be an option; however, the nurse would not just tell the client to do something like that.

A client is admitted with superior vena cava syndrome. What action by the nurse is most appropriate? A. Administer a dose of allopurinol (Aloprim). B. Assess the clients serum potassium level. C. Gently inquire about advance directives. D. Prepare the client for emergency surgery.

C. Gently inquire about advance directives. Rationale: Superior vena cava syndrome is often a late-stage manifestation. After the client is stabilized and comfortable, the nurse should initiate a conversation about advance directives. Allopurinol is used for tumor lysis syndrome. Potassium levels are important in tumor lysis syndrome, in which cell destruction leads to large quantities of potassium being released into the bloodstream. Surgery is rarely done for superior vena cava syndrome.

After teaching a client who is prescribed a long-acting beta2 agonist medication, a nurse assesses the client's understanding. Which statement indicates that the client COMPREHENDS the teaching? a. I will carry this medication with me at all times in case I need it b. I will take this medication when I start to experience an asthma attack c. I will take this medication every morning to help prevent an acute attack d. I will be weaned off this medication when I no longer need it

C. I will take this medication every morning to help prevent an acute attack Long-acting beta2 agonist medications will help prevent an acute asthma attack because they are long acting. The client will take this medication every day for best effect. The client does not have to always keep this medication with him or her because it is not used as a rescue medication. This is not the medication the client will use during an acute asthma attack because it does not have an immediate onset of action. The client will not be weaned off this medication because this is likely to be on of his or her daily medications.

After teaching a client who is prescribed salmeterol, the nurse assesses the client's understanding. Which statement by the client indicates a need for ADDITIONAL teaching? a. I will be certain to shake the inhaler well before I use it b. It may take a while before I notice a change in my asthma c. I will use the drug when I have an asthma attack d. I will be careful not to let the drug escape out of my nose and mouth

C. I will use the drug when I have an asthma attack Salmeterol is a long-acting beta2 agonist designed to prevent an asthma attack; it does not relieve or reverse symptoms. Salmeterol has a slow onset of action; therefore, it would not be used as a rescue drug. The drug must be shaken well because it has a tendency to separate easily. Poor technique on the client's part allows the drug to escape through the nose and mouth.

A nurse cares for a client with chronic obstructive pulmonary disease (COPD). The client states that going out with friends is no longer enjoyable. How would the nurse respond? a. There are a variety of support groups for people who have COPD b. I will ask your primary healthcare provider to prescribe an anti-anxiety agent c. I'd like to hear about thoughts and feelings causing you to limit social activities d. Friends can be a good support system for clients with chronic disorders

C. I'd like to hear about thoughts and feelings causing you to limit social activities Many clients with moderate to severe COPD become socially isolated because they are embarrassed by frequent coughing and mucus production. They also can experience fatigue, which limits their activities. The nurse needs to encourage the client to verbalize thoughts and feelings so that appropriate interventions can be selected. Joining a support group would not decrease feelings of social isolation if the client does not verbalize feelings. Anti-anxiety agents will not help the client with social isolation. While friends can be good sources of support, the client specifically is discussing going out of the home.

A client is receiving rituximab (Rituxan) and asks how it works. What response by the nurse is best? A. It causes rapid lysis of the cancer cell membranes. B. It destroys the enzymes needed to create cancer cells. C. It prevents the start of cell division in the cancer cells. D. It sensitizes certain cancer cells to chemotherapy.

C. It prevents the start of cell division in the cancer cells. Rationale: Rituxan prevents the initiation of cancer cell division. The other statements are not accurate.

A nurse cares for a client who is infected with Burkholderia cepacia. What action would the nurse take FIRST when admitting this client to a pulmonary care unit? a. Instruct the client to wash his or her hands after contact with other people b. Implement Droplet Precautions and don a surgical mask c. Keep the client separated from other clients with cystic fibrosis d. Obtain blood, sputum, and urine culture specimens

C. Keep the client separated from other clients with cystic fibrosis B. cepacia infection is spread through casual contact between cystic fibrosis clients, thus the need for infected clients to be separated from non-infected clients. Strict isolation measures will not be necessary. Although the client would wash his or hands frequently, the most important measure that can be implemented on the unit is isolation of the client from other clients with cystic fibrosis. There is no need to implement Droplet Precautions or don a surgical mask when caring for this client. Obtaining blood, sputum, and urine culture specimens will not provide information necessary to care for a client with B. cepacia infection.

A nurse cares for a client who has developed esophagitis after undergoing radiation therapy for lung cancer. Which diet selection would the nurse provide for this client? a. Spaghetti with meat sauce, ice cream b. Chicken soup, grilled cheese sandwich c. Omelet, soft whole-wheat bread d. Pasta salad, custard, orange juice

C. Omelet, soft whole-wheat bread Side effects of radiation therapy may include inflammation of the esophagus. Clients would be taught that bland, soft, high-calorie foods are best, along with liquid nutritional supplements. Tomato sauce may prove too spicy for a client with esophagitis. A grilled cheese sandwich is too difficult to swallow with this condition, and orange juice and other foods with citric acid are too caustic.

A nurse cares for a client who has a family history of cystic fibrosis. The client asks, "Will my children have CF?" How would the nurse respond? a. Since many of your family members are carriers, your children will also be carriers of the gene b. CF is an autosomal recessive disorder. If you are a carrier, your children will have the disorder c. Since you have a family history of CF, I would encourage you and your partner to be tested d. CF is caused by a protein that controls the movement of chloride. Adjusting your diet will decrease the spread of this disorder

C. Since you have a family history of CF, I would encourage you and your partner to be tested CF is an autosomal recessive disorder in which both gene alleles must be mutated for the disorder to be expressed. The nurse would encourage both the client and partner to be tested for the abnormal gene. The other statements are not true.

The nurse is caring for a client who has cystic fibrosis (CF). The client asks for information about gene therapy. What response by the nurse is BEST? a. Unfortunately, gene therapy is only provided to children upon diagnosis b. Do you know that you will have to get genetic testing? c. There is a good treatment for the most common genetic defect in CF d. Gene therapy will only help improve your pulmonary symptoms

C. There is a good treatment for the most common genetic defect in CF The drug ivacaftor/lumacaftor is effective as therapy for patients whose CF is caused by the F508del (aka the Phe508del) mutation, the most common mutation involved in CF, even in patients who are homozygous for the mutation with both alleles being affected. The nurse would provide that information as the best response. Asking if the client understands he or she will have to undergo genetic testing is a correct statement, but is a yes/no question which is not therapeutic and might sound paternalistic. It also does not provide any information on the therapy itself. The drug is not limited to children and helps move chloride closer to the membrane surfaces so it would have an effect on any organ compromised by CF.

A nurse cares for a client who test positive for alpha1-antitrypsin (AAT) deficiency. The client asks, "What does this mean?" How would the nurse respond? a. Your children will be at high risk for COPD b. I will contact a genetic counselor to discuss your condition c. Your risk for COPD is higher, especially if you smoke d. This is a recessive gene and would have no impact on your health

C. Your risk for COPD is higher, especially if you smoke Alpha1-antitrypsin deficiency is an important risk factor for COPD. The gene for AAT is a recessive gene. Clients with only one allele produce enough AAT to prevent COPD unless the client smokes or there is sufficient exposure to other inhalants. A client with two alleles is at high risk for COPD even if not exposed to smoke or other irritants. The client is a carrier, and children may or may not be at high risk depending on the partner's AAT levels. Contacting a genetic counselor may be helpful but does not address the client's current question.


Conjuntos de estudio relacionados

Our Communities: Where Communities are Located, Three Types of Communities

View Set

Diabetes Insipidus AND SIADH MY NOTES

View Set

psyc 101 exam 3, ch6 (online test)

View Set

Drivers training segment 2 made by me

View Set

§ 15. Цивільна правоздатність і цивільна дієздатність. Обсяг цивільної правосуб'єктності неповнолітніх.

View Set